You are on page 1of 307

ECOS II Midterm Practice

Questions (PART 2)
DISCLAIMER:
If there are discrepancies between our
slides and what was presented to you in
lecture/lab, please refer to your lecture
and lab slides.

For clarifications on specific questions, please


reach out to the tutor who wrote the question.
Lecture Questions
Ribs

Lamia Faruk and Victor Phan


1. What type of Muscle Energy Principle do we use to perform Ribs OMT?

A. Isokinetic Strengthening and Post-Isometric Relaxation


B. Respiratory Assistance and Post-Isometric Relaxation
C. Joint Mobilization Using Muscle Force and Crossed Extensor Reflex
D. Respiratory Assistance and Joint Mobilization Using Muscle Force
1. What type of Muscle Energy Principle do we use to perform Ribs OMT?

A. Isokinetic Strengthening and Post-Isometric Relaxation


B. Respiratory Assistance and Post-Isometric Relaxation
C. Joint Mobilization Using Muscle Force and Crossed Extensor Reflex
D. Respiratory Assistance and Joint Mobilization Using Muscle Force
This questions stem will be used in the next two questions

2. A 40 year old obese male patient arrives to your clinic for a check up exam. He has a 10
pack year of smoking, which you have counseled him about in the past. However, you notice
today that he his coughing frequently during your exam. He tells you that he’s been short of
breath whenever he tries to walk, and you notice that he appears to be breathing through his
lips. You perform an OMT exam, and you note that ribs 3-6 does not move superiorly with
inhalation on the right as compared to the ribs on the left. What type of somatic dysfunction
does he have?

A. Left Ribs 3-5 Exhalation SD


B. Right Ribs 3-5 Exhalation SD
C. Left Ribs 3-5 Inhalation SD
D. Right Ribs 3-5 Exhalation SD.
This questions stem will be used in the next two questions
A 40 year old obese male patient arrives to your clinic for a check up exam. He has a 10 pack year of smoking, which you
have counseled him about in the past. However, you notice today that he his coughing frequently during your exam. He tells
you that he’s been short of breath whenever he tries to walk, and you notice that he appears to be breathing through his
lips. You perform an OMT exam, and you note that ribs 3-6 does not move superiorly with inhalation on the right as
compared to the ribs on the left. What type of somatic dysfunction does he have?
A. Left Ribs 3-5 Exhalation SD
B. Right Ribs 3-5 Exhalation SD
C. Left Ribs 3-5 Inhalation SD
D. Right Ribs 3-5 Exhalation SD.
This questions stem will be used in the next two questions

3. A 40 year old obese male patient arrives to your clinic for a check up exam. He has a 10
pack year of smoking, which you have counseled him about in the past. However, you notice
today that he his coughing frequently during your exam. He tells you that he’s been short of
breath whenever he tries to walk, and you notice that he appears to be breathing through his
lips. You perform an OMT exam, and you note that ribs 3-6 does not appear to move up much
on the right as it does on the left. Which rib would you treat first?

A. Right Rib 6
B. Right Rib 5
C. Right Rib 3
D. Left Rib 3
This questions stem will be used in the next two questions
3. A 40 year old obese male patient arrives to your clinic for a check up exam. He has a 10 pack year of smoking, which you have
counseled him about in the past. However, you notice today that he his coughing frequently during your exam. He tells you that he’s
been short of breath whenever he tries to walk, and you notice that he appears to be breathing through his lips. You perform an
OMT exam, and you note that ribs 3-6 does not appear to move up much on the right as it does on the left. Which rib would you
treat first?
A. Right Rib 6
B. Right Rib 5
C. Right Rib 3
D. Left Rib 3
4. A 25 year old female medical student comes to your clinic complaining of neck pain. She
says that she has been doing a lot of hunching over and studying in the past few days, but
she noted that her neck pain seemed to have gotten worse after she tripped and fell
forward while running to class. You perform an osteopathic examination on her cervical
pain and note no asymmetry, but upon examination of her chest, you not that Rib 2 on her
right chest does not seem to move superiorly in inhalation as compared to her left. Which
muscle would you use to treat this somatic dysfunction?

A. Middle Scalene
B. Anterior Scalene
C. Posterior Scalene
D. Anterior + Middle Scalene
4. A 25 year old female medical student comes to your clinic complaining of neck pain. She says that she
has been doing a lot of hunching over and studying in the past few days, but she noted that her neck pain
seemed to have gotten worse after she tripped and fell forward while running to class. You perform an
osteopathic examination on her cervical pain and note no asymmetry, but upon examination of her chest,
you not that Rib 2 on her right chest does not seem to move superiorly in inhalation as compared to her
left. Which muscle would you use to treat this somatic dysfunction?
A. Middle Scalene
B. Anterior Scalene
C. Posterior Scalene
D. Anterior + Middle Scalene
5. What is the motion of ribs 1-2, 3-7, 8-10, and 11-12 respectively?

A. Caliper, Pump Handle, Pump Handle, Bucket Handle


B. Bucket Handle, Pump Handle, Bucket Handle, Caliper
C. Pump Handle, Bucket Handle, Pump Handle, Caliber
D. Bucket Handle, Bucket Handle, Pump Handle, Pump Handle
5. What is the motion of ribs 1-2, 3-7, 8-10, and 11-12 respectively?

A. Caliper, Pump Handle, Pump Handle, Bucket Handle


B. Bucket Handle, Pump Handle, Bucket Handle, Caliper
C. Pump Handle, Bucket Handle, Pump Handle, Caliber
D. Bucket Handle, Bucket Handle, Pump Handle, Pump Handle
GOOD LUCK ON YOUR MIDTERM Y’ALL!

If you have any questions, feel free to email me at farhana.faruk@kansascity.edu!!


Question 1

When palpating the thoracic region of your friend, you realize that there is a PTP on the right of T4.
Remembering your rib anatomy, which of the following statements is correct in regards to the
articulation of the rib and the spine?

A. The inferior costal facet of T4 is displaced from the head of the 3rd rib
B. The inferior costal facet of T3 is displaced from the head of the 4th rib
C. The superior costal facet of T3 is displaced from the head of the 4th rib
D. The inferior costal facet of T4 is displaced from the head of the 4th Rib
Q1: Answer
When palpating the thoracic region of your friend, Know what articulates and its anatomy with what!
you realize that there is a PTP on the right of T4. There were a few questions on this
Remembering your rib anatomy, which of the
following statements is correct in regards to the Rib 4 with the superior costal of T4 and the
articulation of the rib and the spine?
inferior costal of T3
A. The inferior costal facet of T4 is displaced
from the head of the 3rd rib
B. The inferior costal facet of T3 is displaced
from the head of the 4th rib
C. The superior costal facet of T3 is displaced
from the head of the 4th rib
D. The inferior costal facet of T4 is displaced
from the head of the 4th Rib
Question 2: Matching

Rib 1 No Cartilaginous Attachment

Rib 10 Cartilage Attaches to the sternum

Rib 12 Cartilage attaches to the rib above


Question 2: Answer

Rib 1 No Cartilaginous Attachment

Rib 10 Cartilage Attaches to the sternum

Rib 12 Cartilage attaches to the rib above

LO 1: All about the anatomy


Question 3

You are in lab with a fellow classmate that presents with slight back pain. Upon evaluation, you see that
there is a SD with T4. You also find that there is a rib somatic dysfunction that resists inhalation for the
corresponding rib and all the ones above it. What would the proper MET technique be?

A. Pushing on Rib 4 Posteriorly


B. A high velocity force on T4
C. Using the latissimus dorsi to improve the rib in exhalation
D. Using the pectoralis minor to improve the rib in inhalation
Question 3: Answer LO 2 and 3

You are in lab with a fellow classmate that presents with slight back pain. Upon evaluation, you see that
there is a SD with T4. You also find that there is a rib somatic dysfunction that resists inhalation for the
corresponding rib and all the ones above it. What would the proper MET technique be?

A. Depress Rib 3 with respiratory resistance


B. A high velocity force on T4
C. Using the pectoralis minor to improve the rib in exhalation
D. Using the anterior scalene to improve the rib in inhalation
a. Remember that the ribs articulate with the spine: Same number with the inferior facet and one above with
the superior facet
b. This is an exhalation somatic dysfunction
c. BITE: Treat the top one for exhalation: Because all the ones above it were affected, you treat rib 1
d. Rib 1 MET is performed with the anterior AND middle scalene
Question 4

Which of the following is NOT true about rib anatomy?

A. Ribs 11 and 12 are false ribs


B. Ribs 1-7 are True ribs
C. Ribs 8-10 are false ribs
D. The costochondral joint attaches the rib to the sternum
E. The tubercle of the rib articulates with the transverse process
Question 4: LO1

Which of the following is NOT true about rib anatomy?

A. Ribs 11 and 12 are false ribs


B. Ribs 1-7 are True ribs
C. Ribs 8-10 are false ribs
D. The costochondral joint attaches the rib to the sternum
a. This is false. The costochondral joint is the attachment of cartilage to the rib
b. On the other side of the cartilage is the sternocostal joint that attaches it to the sternum
E. The tubercle of the rib articulates with the transverse process
Question 5

Which of the following is true about treating Ribs 4-8 Exhalation Somatic Dysfunction?

A. When treating, you will want to treat rib 8 and the rest will follow
B. When treating you will use pectoralis minor
C. When treating you will elevate rib for with respiratory assistance
D. When treating you will use the latissimus dorsi
Question 5: LO3

Which of the following is true about treating Ribs 4-8 Exhalation Somatic Dysfunction?

A. When treating, you will want to treat rib 8 and the rest will follow
B. When treating you will use pectoralis minor
a. The key rib is rib 4. According to BITE, you treat the top rib
b. In an exhalation somatic dysfunction you use the muscles to treat
c. Rib 4 muscle is the pectoralis minor
d. Rib 8 is the serratus anterior BUT rib 8 is NOT the key rib
C. When treating you will elevate rib for with respiratory assistance
D. When treating you will use the serratus anterior
If you have any questions please reach out!
Victor.Phan@kansascity.edu
Approach to Cardiac
Complaint
Tyler Emery & Hannah Riga
Approach to Cardiac Complaint

1) A grade III/VI systolic murmur is located on the cardiac exam. The murmur is loudest at the right
2nd intercostal space. Which is the most likely murmur pathology?
a) Aortic Regurgitation
b) Aortic stenosis
c) Mitral regurgitation
d) Pulmonary stenosis
Approach to Cardiac Complaint

1) A grade III/VI systolic murmur is located on the cardiac exam. The murmur is loudest at the right
2nd intercostal space. Which is the most likely murmur pathology?
a) Aortic Regurgitation-correct location but diastolic murmur
b) Aortic stenosis
c) Mitral regurgitation-systolic murmur but wrong location
d) Pulmonary stenosis-systolic murmur but wrong location
Approach to Cardiac Complaint

2) A systolic murmur will be located between which heart sounds?

a) S2-S1
b) S1-S2
c) S4-S1
d) S2-S3
Approach to Cardiac Complaint

2) A systolic murmur will be located between which heart sounds?

a) S2-S1-diastolic murmur
b) S1-S2
c) S4-S1
d) S2-S3

Its important to recognize which murmurs fall between which heart sounds and where they fall within the
cardiac cycle.
Approach to Cardiac Complaint

3) A laterally displaced PMI is most likely to indicate what?

a) Left ventricular hypertrophy


b) Aortic regurgitation
c) Elevated JVP
d) Myocardial infarction
Approach to Cardiac Complaint

3) A laterally displaced PMI is most likely to indicate what?

a) Left ventricular hypertrophy-this makes sense because a larger ventricle will displace that big LV
contraction further laterally. Look for a possible x-ray with cardiomegaly and a correlation to what
PE finding you may fine
b) Aortic regurgitation
c) Elevated JVP
d) Myocardial infarction
Approach to Cardiac Complaint

4) What is the first grade for murmurs that is associated with a thrill?

a) I
b) II
c) III
d) IV
e) V
f) VI
Approach to Cardiac Complaint

4) What is the first grade for murmurs that is associated with a thrill?

a) I
b) II
c) III
d) IV
e) V
f) VI
Approach to Cardiac Complaint

5) Which CVP wave finding is found with tricuspid regurgitation?

a) Prominent V wave
b) Decreased V wave
Approach to Cardiac Complaint

5) Which CVP/jugular venous wave finding is found with tricuspid regurgitation?

a) Prominent V wave
b) Decreased V wave

Think about why this is… If there is backflow of blood during RV systole, you will see the spike in the
corresponding ‘v’ wave. Ask yourself what findings would be seen in a stenotic tricuspid valve.
Contact me with any questions

tyler.emery@kansascity.edu
Approach to Cardiac Complaint

6) While doing third year rotations as a medical student, you observe a cardiologist place a patient into a
left lateral decubitus position and then auscultate the left lower heart. You ask the cardiologist why they
have placed the patient in this position and they answer with:

a) To listen for high-pitched murmurs


b) To listen for aortic regurgitation
c) To listen for low-pitched extra heart sounds (S3, S4) and low-pitched murmurs (mitral stenosis)
d) To prevent breath sounds from obstructing the heart sounds
Approach to Cardiac Complaint

6) While doing third year rotations as a medical student, you observe a cardiologist place a patient into a
left lateral decubitus position and then auscultate the left lower heart. You ask the cardiologist why they
have placed the patient in this position and they answer with:

a) To listen for high-pitched murmurs


b) To listen for aortic regurgitation
c) To listen for low-pitched extra heart sounds (S3, S4) and low-pitched murmurs (mitral stenosis)
d) To prevent breath sounds from obstructing the heart sounds
Approach to Cardiac Complaint

7) While doing third year rotations as a medical student, you observe a cardiologist listen at all four
listening posts with the large rounded side of their stethoscope, then switch to the smaller rounded side,
and then they finally proceeded to auscultate the heart again at the same four listening posts. You ask the
cardiologist what they used the smaller rounded side for, they respond with:

a) To listen for low-pitched sounds (S3 and S4, and mitral stenosis murmur)
b) To listen for high-pitched sounds (S1 and S2, and most murmurs)
c) To listen for aortic regurgitation
d) To prevent breath sounds from obstructing heart sounds
Approach to Cardiac Complaint

7) While doing third year rotations as a medical student, you observe a cardiologist listen at all four
listening posts with the large rounded side of their stethoscope, then switch to the smaller rounded side,
and then they finally proceeded to auscultate the heart again at the same four listening posts. You ask the
cardiologist what they used the smaller rounded side for, they respond with:

a) To listen for low-pitched sounds (S3 and S4, and mitral stenosis murmur)
b) To listen for high-pitched sounds (S1 and S2, and most murmurs)
c) To listen for aortic regurgitation
d) To prevent breath sounds from obstructing heart sounds
Approach to Cardiac Complaint

8) While completing a thorough physical exam on a patient, you palpate their chest wall. While doing so,
you feel a buzzing/vibratory sensation. What is this indicative of?

a) Tenderness of the chest wall


b) Heave
c) Thrill
d) Regular heart beat
Approach to Cardiac Complaint

8) While completing a thorough physical exam on a patient, you palpate their chest wall. While doing so,
you feel a buzzing/vibratory sensation. What is this indicative of?

a) Tenderness of the chest wall


b) Heave
c) Thrill
d) Regular heart beat
Approach to Cardiac Complaint

9) Which of the following is associated with the INCORRECT listening post:

a) Aortic - right 4th intercostal space at sternal border


b) Pulmonic - left 2nd intercostal space at sternal border
c) Tricuspid - left 4th intercostal space at sternal border
d) Mitral - left 5th intercostal space at mid-clavicular line
Approach to Cardiac Complaint

9) Which of the following is associated with the INCORRECT listening post:

a) Aortic - right 4th intercostal space at sternal border


b) Pulmonic - left 2nd intercostal space at sternal border
c) Tricuspid - left 4th intercostal space at sternal border
d) Mitral - left 5th intercostal space at mid-clavicular line
My favorite mnemonic to remember the listening posts is “All Physicians Take Money”
or if that bothers you another one is “All Patients Take Meds”
Approach to Cardiac Complaint

10) You are still rotating with that same cardiologist during your third year rotations. They describe a
patient to you that is experiencing significant lower extremity edema (+3/4) with an elevated JVP of 9cm.
They ask you what is the most likely underlying cardiac etiology causing this patient’s signs/symptoms,
you reply with:

a) Aortic stenosis
b) Hypertrophic cardiomyopathy
c) Hypertension
d) Congestive heart failure
Approach to Cardiac Complaint

10) You are still rotating with that same cardiologist during your third year rotations. They describe a
patient to you that is experiencing significant lower extremity edema (+3/4) with an elevated JVP of 9cm.
They ask you what is the most likely underlying cardiac etiology causing this patient’s signs/symptoms,
you reply with:

a) Aortic stenosis
b) Hypertrophic cardiomyopathy
c) Hypertension
d) Congestive heart failure
Contact me with any questions.
Good luck!
Hannah Riga
hannah.riga@kansascity.edu
Approach to Cardiac Sounds
& Murmurs
Minah Tariq and Andrea Picciotto
1. Arrange the following phases of the cardiac cycle in order, from the end of diastole to the start of systole.

1. Isovolumetric relaxation
2. Rapid ejection
3. Reduced filling
4. Reduced ejection
5. Atrial contraction
6. Isovolumetric contraction
7. Rapid filling

1. 4, 7, 2, 3, 1, 6, 5
2. 5, 6, 2, 4, 1, 7, 3
3. 1, 2, 3, 4 , 6, 5, 7
4. 5, 6, 4, 2, 1, 7, 3
1. Arrange the following phases of the cardiac cycle in order, from the end of diastole to the start of systole.

1. Isovolumetric relaxation
2. Rapid ejection
3. Reduced filling
4. Reduced ejection
5. Atrial contraction
6. Isovolumetric contraction
7. Rapid filling

1. 4, 7, 2, 3, 1, 6, 5
2. 5, 6, 2, 4, 1, 7, 3
3. 1, 2, 3, 4 , 6, 5, 7
4. 5, 6, 4, 2, 1, 7, 3
2. You’re on your cardio rotations and your preceptor asks you to answer some basic questions about cardiac
physiology and to interpret a few heart sounds, no big deal. They ask you to explain why the S2 splitting sound can
be heard with some patients. You tell them…

a. Because patients with an abnormally large heart will have a split S2 sound
b. Because the “dub” sound is due to Aortic and Pulmonic valves closing 1 leaflet at a time
c. S2 splitting can be heard if blood is hitting a hard ventricular wall
d. Because it takes longer for the right ventricle to squeeze the extra blood and for the pulmonary
valve to close
2. You’re on your cardio rotations and your preceptor asks you to answer some basic questions about cardiac
physiology and to interpret a few heart sounds, no big deal. They ask you to explain why the S2 splitting sound can
be heard with some patients. You tell them…

a. Because patients with an abnormally large heart will have a split S2 sound
b. Because the “dub” sound is due to Aortic and Pulmonic valves closing 1 leaflet at a time
c. S2 splitting can be heard if blood is hitting a hard ventricular wall
d. Because it takes longer for the right ventricle to squeeze the extra blood and for the
pulmonary valve to close
3. A trained 19 year old athlete comes to your outpatient clinic for a regular checkup. They have had no symptoms but
when you listen to their heart, there is an interesting third noise heard on auscultation. What is the best position to
ask your patient to stay still in so that this third sound can be heard best?

a. Sitting up on the table with their feet touching the ground and their palms resting on their legs
b. Patient lying down in left lateral decubitus
c. Patient lying down in right lateral decubitus
d. Patient supine on table
3. A trained 19 year old athlete comes to your outpatient clinic for a regular checkup. They have had no symptoms but
when you listen to their heart, there is an interesting third noise heard on auscultation. What is the best position to
ask your patient to stay still in so that this third sound can be heard best?

a. Sitting up on the table with their feet touching the ground and their palms resting on their legs
b. Patient lying down in left lateral decubitus
c. Patient lying down in right lateral decubitus
d. Patient supine on table
4. When studying the heart’s physiology for your super easy ECOSII exam, it’s good to remember the basics and build
a solid foundation. For example, it’s good to know that during _________, the ventricles are relaxed and at that time,
the AV valves are _______ to let blood flow through the atria and ventricles.

a. Systole, open
b. Diastole, closed
c. Diastole open
d. Systole, closed
4. When studying the heart’s physiology for your super easy ECOSII exam, it’s good to remember the basics and build
a solid foundation. For example, it’s good to know that during _________, the ventricles are relaxed and at that time,
the AV valves are _______ to let blood flow through the atria and ventricles.

a. Systole, open
b. Diastole, closed
c. Diastole open
d. Systole, closed
5. It’s time to show your standardized patient that you know how to properly auscultate them for heart sounds! When
listening with the diaphragm of your stethoscope on bare skin, where is the best position to hear a Mitral valve
murmur?

a. Left 4th intercostal space at sternal border


b. Right 2nd intercostal space at sternal border
c. Left 2nd intercostal space at sternal border
d. Left 5th intercostal space at mid-clavicular line
5. It’s time to show your standardized patient that you know how to properly auscultate them for heart sounds! When
listening with the diaphragm of your stethoscope on bare skin, where is the best position to hear a Mitral valve
murmur?

a. Left 4th intercostal space at sternal border - Tricuspid valve


b. Right 2nd intercostal space at sternal border - Aortic valve
c. Left 2nd intercostal space at sternal border - Pulmonic valve
d. Left 5th intercostal space at mid-clavicular line
Contact me with any questions!

minah.tariq@kansascity.edu
Where would you place your stethoscope to listen to
the pulmonic valve?
a. Left 2nd intercostal space at the sternal border
b. Right 2nd intercostal space at sternal border
c. Left 3rd intercostal space at sternal border
d. Left 5th intercostal space at mid-clavicular line
Where would you place your stethoscope to listen to
the pulmonic valve?
a. Left 2nd intercostal space at the sternal border Aortic
b. Right 2nd intercostal space at sternal border
c. Left 3rd intercostal space at sternal border
d. Left 5th intercostal space at mid-clavicular line Mitral
What happens during inspiration?

a. S2 gets louder at the aortic and pulmonic posts, diaphragm raises, increase in venous blood to the
right atrium and to the right ventricle
b. S2 splits, diaphragm lowers, decrease in venous blood to the right atrium and to the right ventricle
c. S3 splits, diaphragm relaxes to dome shape, decrease in venous blood to the left atrium and to the
left ventricle
d. S1 gets louder at the tricuspid and mitral valve, diaphragm raises, increase in venous blood to the
right atrium and to the right ventricle
What happens during inspiration?

a. S2 gets louder at the aortic and pulmonic posts, diaphragm raises, increase in venous blood to the
right atrium and to the right ventricle
b. S2 splits, diaphragm lowers, decrease in venous blood to the right atrium and to the right
ventricle
c. S3 splits, diaphragm relaxes to dome shape, decrease in venous blood to the left atrium and to the
left ventricle
d. S1 gets louder at the tricuspid and mitral valve, diaphragm raises, increase in venous blood to the
right atrium and to the right ventricle
Label the four different types of S2 splitting in order from A-D

A
A. Normal, Fixed Splitting, Paradoxical Splitting, Wide Splitting

B. Normal, Wide Splitting, Fixed Splitting, Paradoxical Splitting B


C. Normal, Fixed Splitting, Wide Splitting, Paradoxical Splitting

D. Paradoxical Splitting, Wide splitting, Fixed Splitting, Normal


C

D
Label the four different types of S2 splitting in order from A-D

A
A. Normal, Fixed Splitting, Paradoxical Splitting, Wide Splitting

B. Normal, Wide Splitting, Fixed Splitting, Paradoxical Splitting B


C. Normal, Fixed Splitting, Wide Splitting, Paradoxical Splitting

D. Paradoxical Splitting, Wide splitting, Fixed Splitting, Normal


C

D
Name the diastolic murmurs

a. aortic/pulmonic stenosis, mitral/tricuspid regurgitation


b. aortic/pulmonic regurgitation, mitral and tricuspid stenosis
c. Patent Ductus arteriosus, aortic stenosis, tricuspid regurgitation
d. Aortic stenosis and regurgitation
Name the diastolic murmurs

a. aortic/pulmonic stenosis, mitral/tricuspid regurgitation SYSTOLIC


b. aortic/pulmonic regurgitation, mitral and tricuspid stenosis
c. Patent Ductus arteriosus, aortic stenosis, tricuspid regurgitation
d. Aortic stenosis and regurgitation
Your patient has a murmur that is louder in volume
than S1 and S2 with a palpable thrill. What is the
grade of this murmur?
a. Grade 3/6
b. Grade 4/6
c. Grade ⅚
d. Grade 6/6
Your patient has a louder in volume than S1 and S2
with a palpable thrill. What is the grade of this
murmur?
a. Grade 3/6
b. Grade 4/6
c. Grade ⅚
d. Grade 6/6
Email me with any questions!

andrea.picciotto@kansascity.edu

Goodluck!
Approach to Lower
Respiratory Complaint
Marenda Biggs & Hannah Riga
Approach to Lower Respiratory Complaint

1) During your residency, a patient under your care needs a chest tube placement. Your attending
reminds you to place it over the superior margin of the rib. Why is this necessary?
a) The tissue is softer in this area, allowing for the tube to go in easier
b) To avoid any neurovasculature
c) There are less pain receptors in this area, so it will be less painful to the patient
d) This is just standard protocol, there is no actual reason to do this
Approach to Lower Respiratory Complaint

1) During your residency, a patient under your care needs a chest tube placement. Your attending
reminds you to place it over the superior margin of the rib. Why is this necessary?
a) The tissue is softer in this area, allowing for the tube to go in easier
b) To avoid any neurovasculature
c) There are less pain receptors in this area, so it will be less painful to the patient
d) This is just standard protocol, there is no actual reason to do this
Approach to Lower Respiratory Complaint

2) During your fourth year rotations, you rotate in the ICU and observe a patient with an abnormal
breath pattern. They are tachypneic, with gradual hyperpnea, then hypopnea, followed by a period of
periodic apnea. What is the likely underlying condition causing this patient’s breathing pattern?

a) Heart failure
b) Diabetic ketoacidosis
c) Severe atelectasis following surgery
d) Obstructive sleep apnea
e) Chronic obstructive pulmonary disease
Approach to Lower Respiratory Complaint

2) During your fourth year rotations, you rotate in the ICU and observe a patient with an abnormal
breath pattern. They are tachypneic, with gradual hyperpnea, then hypopnea, followed by a period of
periodic apnea. What is the likely underlying condition causing this patient’s breathing pattern?

a) Heart failure
b) Diabetic ketoacidosis
c) Severe atelectasis following surgery
d) Obstructive sleep apnea
e) Chronic obstructive pulmonary disease
Approach to Lower Respiratory Complaint

3) During your fourth year rotations, you rotate in the pediatric ICU and observe a patient with an
abnormal breath pattern. They have deep, rapid, and labored breathing. What is the likely underlying
condition causing this patient’s breathing pattern?

a) Heart failure
b) Diabetic ketoacidosis
c) Severe atelectasis following surgery
d) Obstructive sleep apnea
e) Chronic obstructive pulmonary disease
Approach to Lower Respiratory Complaint

3) During your fourth year rotations, you rotate in the pediatric ICU and observe a patient with an
abnormal breath pattern. They have deep, rapid, and labored breathing. What is the likely underlying
condition causing this patient’s breathing pattern?

a) Heart failure
b) Diabetic ketoacidosis
c) Severe atelectasis following surgery
d) Obstructive sleep apnea
e) Chronic obstructive pulmonary disease
Approach to Lower Respiratory Complaint

4) During your fourth year rotations, you rotate in the ICU and observe a patient with shortness of breath
and coughing. Your preceptor tells you that the patient has fluid within their pleural space. They also
show you the patient’s chest x-ray, which shows blunting of the right costophrenic angle and fluid within
the right oblique fissure. What is the most likely underlying infectious condition causing this patient’s
signs/symptoms?

a) Congestive heart failure


b) Pneumonia
c) Malignancy
d) Pulmonary embolism
Approach to Lower Respiratory Complaint

4) During your fourth year rotations, you rotate in the ICU and observe a patient with shortness of breath
and coughing. Your preceptor tells you that the patient has fluid within their pleural space. They also
show you the patient’s chest x-ray, which shows blunting of the right costophrenic angle and fluid within
the right oblique fissure. What is the most likely underlying infectious condition causing this patient’s
signs/symptoms?
*Note: all of these can
a) Congestive heart failure cause pleural effusion,
b) Pneumonia however pneumonia is the
only infectious etiology,
c) Malignancy remember to read the
d) Pulmonary embolism question carefully.
Approach to Lower Respiratory Complaint

5) During a physical exam, you are auscultating a patient’s breath sounds and notice a shrill, high-pitched
whistling sound that is mostly expiratory. The patient states they also hear this breathing sound, and that
it gets worse during spring and fall seasons. The patient has a medical history of eczema and seasonal
allergies. What is the most likely underlying condition causing this patient’s abnormal breath sounds?

a) Pertussis
b) Croup
c) Pneumothorax
d) Congestive heart failure
e) Asthma
Approach to Lower Respiratory Complaint

5) During a physical exam, you are auscultating a patient’s breath sounds and notice a shrill, high-pitched
whistling sound that is mostly expiratory. The patient states they also hear this breathing sound, and that
it gets worse during spring and fall seasons. The patient has a medical history of eczema and seasonal
allergies. What is the most likely underlying condition causing this patient’s abnormal breath sounds?

a) Pertussis - barking cough > high-pitched “whoop” sound


b) Croup - stridor
c) Pneumothorax - decreased/absent breath sounds
d) Congestive heart failure - crackles
e) Asthma
Contact me with any questions.
Good luck!
Hannah Riga
hannah.riga@kansascity.edu
A 6-year old boy is brought to clinic with his mom after she notices he makes a high pitched sound when he breathes in.
Upon physical examination, you note the sound to be croup. What is a possible cause for this?
A. Pneumothorax
B. Pneumonia
C. Epiglottitis
D. Asthma attack
A 6-year old boy is brought to clinic with his mom after she notices he makes a high pitched sound when he breathes in.
Upon physical examination, you note the sound to be croup. What is a possible cause for this?
A. Pneumothorax
B. Pneumonia
C. Epiglottitis
D. Asthma attack
A 65- year old female presents to clinic with complaints of shortness of breath. Upon further testing, you diagnose her
with COPD. What is the expected breath sound to hear on auscultation?

A. Wheezing
B. Crackles
C. CTAB
D. Stridor
A 65- year old female presents to clinic with complaints of shortness of breath. Upon further testing, you diagnose her
with COPD. What is the expected breath sound to hear on auscultation?

A. Wheezing
B. Crackles
C. CTAB
D. Stridor

A 65- year old female presents to clinic with complaints of shortness of breath. Upon general examination, you
note that she is learning forward with her elbows resting on her knees for the majority of the visit. What does
this general finding suggest about what you will hear on auscultation during physical exam?
A. Wheezing
B. Crackles
C. CTAB
D. Stridor
During your clinical rotations, you hear the attending pulmonologist use the terms “rhonchi” and “rales.” You immediately
begin to panic because you do not know these terms despite studying hard the weeks before this rotation. You ask your
fellow medical students and they re-assure you that you know what those breath sounds are and what they indicate, but
that the attending just used a different name than what you studied. What are other names for these two terms?
A. Rhonchi: a low pitched wheeze
B. Rhonchi: a high pitched wheeze
C. Rales: another name for crackles
D. Rales: another name for stridor
E. A and C are correct
F. B and D are correct
During your clinical rotations, you hear the attending pulmonologist use the terms “rhonchi” and “rales.” You immediately
begin to panic because you do not know these terms despite studying hard the weeks before this rotation. You ask your
fellow medical students and they re-assure you that you know what those breath sounds are and what they indicate, but
that the attending just used a different name than what you studied. What are other names for these two terms?
A. Rhonchi: a low pitched wheeze
B. Rhonchi: a high pitched wheeze
C. Rales: another name for crackles
D. Rales: another name for stridor
E. A and C are correct
F. B and D are correct
A 65-year old male presents to the clinic with difficulty breathing and lower extremity edema. Upon examination, you note
a “popping” during inspiration. Which of the following diagnoses would correlate with that auscultatory finding?
A. Empyema
B. Pneumothorax
C. Pulmonary embolism
D. CHF
A 65-year old male presents to the clinic with difficulty breathing and lower extremity edema. Upon examination, you note
a “popping” during inspiration. Which of the following diagnoses would correlate with that auscultatory finding?
A. Empyema
B. Pneumothorax
C. Pulmonary embolism
D. CHF
Patient presents to ER with difficulty breathing. Upon radiographic imaging, you learn that this patient has a pleural
effusion. Your attending decides this is a great teaching experience and asks you to explain the difference between
transudative and exudative pleural effusion to the group of medical students. What would you say?
A. Exudative occurs when fluid leaks from the blood vessels due to a hydrostatic pressure change.
B. Transudative occurs when there is inflammation of the pleura and lung
C. Exudative occurs when there is inflammation of the pleura and lung
D. Transudative and exudative occur via the same physiologic mechanism. The only difference is the auscultatory
finding.
Patient presents to ER with difficulty breathing. Upon radiographic imaging, you learn that this patient has a pleural
effusion. Your attending decides this is a great teaching experience and asks you to explain the difference between
transudative and exudative pleural effusion to the group of medical students. What would you say?
A. Exudative occurs when fluid leaks from the blood vessels due to a hydrostatic pressure change.
B. Transudative occurs when there is inflammation of the pleura and lung
C. Exudative occurs when there is inflammation of the pleura and lung
D. Transudative and exudative occur via the same physiologic mechanism. The only difference is the auscultatory
finding.
Good luck! As always, feel free to email any
questions you may have.
Marenda Biggs

marenda.biggs@kansascity.edu
Approach to Renal Complaint

Tyler Emery

Sam Heywood
Approach to Renal Complaint

1) Which of the following is most likely to lead to a pre-renal AKI?


a) Uncontrolled type II diabetes
b) Vomiting and diarrhea
c) Ureterolithiasis
d) Benign prostatic hyperplasia
Approach to Renal Complaint

1) Which of the following is most likely to lead to a pre-renal AKI?


a) Uncontrolled type II diabetes-this tends to damage glomeruli and cause intrinsic AKI
b) Vomiting and diarrhea-the loss of volume will lead to a dehydrated status and decreased volume entering the
glomerulus
c) Ureterolithiasis-this is downstream of the kidney and will cause backup of urine
d) Benign prostatic hyperplasia-also downstream to the kidney and can cause excessive storage of urine in the
bladder
Approach to Renal Complaint

2) What does the AEIOU acronym stand for in regards to indications for starting dialysis?
Approach to Renal Complaint

2) What does the AEIOU acronym stand for?

a) Acidosis-clues to this could be in a ABG (metabolic acidosis..)


b) Electrolyte disturbance-look for this in any BMP information given to you
c) Ingestion-look for the scenario of someone attempting suicide by ingesting antifreeze (ethylene
glycol)
d) Volume overload-think of PE findings that could indicate volume overload (CHF exacerbation,
elevated CVP..)
e) Uremia-also think of PE findings (asterixis, uremic frost, etc)
Approach to Renal Complaint

3) A 48 year old male presents to the ED with severe flank pain and altered mental status. A foley
catheter is inserted to collect a urine sample and only 20mL is emptied and there is gross hematuria. You
suspect nephrolithiasis. What is the gold standard for diagnosis of this condition?

a) Ultrasound
b) Non-contrast CT
c) MRI
d) X-ray
Approach to Renal Complaint

3) A 48 year old male presents to the ED with severe flank pain and altered mental status. A foley
catheter is inserted to collect a urine sample and only 20mL is emptied and there is gross hematuria. You
suspect nephrolithiasis. What is the gold standard for diagnosis of this condition?

a) Ultrasound-while this is good for initial screening for hydronephrosis and is good for screening in
pregnant pts, it is not the gold standard
b) Non-contrast CT
c) MRI-too long, too expensive
d) X-ray-good initial screening of the abdomen but not essentially for definitive diagnosis of stones
Approach to Renal Complaint

4) A 60 year old male presents to your office with a 3 day history of unilateral pain, dysuria, and polyuria.
He says he does not do well with his water intake and this is some of the worst pain he has experienced.
You suspect and eventually diagnose nephrolithiasis. Which composition of the stone are you most likely
to see?

a) Calcium oxalate
b) Uric acid
c) Struvite
d) cystine
Approach to Renal Complaint

4) A 60 year old male presents to your office with a 3 day history of unilateral pain, dysuria, and polyuria.
He says he does not do well with his water intake and this is some of the worst pain he has experienced.
You suspect and eventually diagnose nephrolithiasis. Which composition of the stone are you most likely
to see?

a) Calcium oxalate-most commonly seen


b) Uric acid-look for presentation of a patient with gout or is in treatment for cancer
c) Struvite-clue into this if the stem mentions anything about ammonia producing organisms isolated
on culture
d) Cystine-look for this with a very prominent PFHx and a younger onset of disease
Approach to Renal Complaint

5) What are the most common etiologies for CKD?

a) Nephrolithiasis
b) BPH
c) Hypertension and Diabetes
d) glomerulonephritis
Approach to Renal Complaint

5) What are the most common etiologies for CKD?

a) Nephrolithiasis
b) BPH
c) Hypertension and Diabetes
d) glomerulonephritis
tyler.emery@kansascity.edu
1. Define the following renal terms:

A. Proteinuria

B. Glucosuria

C. Hematuria

D. Albuminuria

E. Dysuria

F. Polyuria

G. Uremia
1. Define the following renal terms:

A. Proteinuria - protein in urine

B. Glucosuria - glucose in urine

C. Hematuria - blood in urine

D. Albuminuria - albumin in urine (more specific proteinuria)

E. Dysuria - painful urination

F. Polyuria - frequent urination

G. Uremia - elevated levels of BUN (blood-urea nitrogen)


2. Patient comes in today noticing redness in their urine and has a foamy appearance
that just started last week. What kind of kidney disease would cause these findings?

A. Pre-renal AKI

B. Intrinsic AKI

C. Post-renal AKI

D. Chronic kidney disease (CKD)


2. Patient comes in today noticing redness in their urine and has a foamy appearance
that just started last week. What kind of kidney disease would cause these findings?

A. Pre-renal AKI

B. Intrinsic AKI - hematuria and proteinuria, only has been going on for a week (CKD is > 3 months)

C. Post-renal AKI

D. Chronic kidney disease (CKD)


3. What is the glomerular filtration rate that would indicate kidney
failure/end-stage kidney disease?

A. > 90

B. 45-59

C. 15-29

D. < 15
3. What is the glomerular filtration rate that would indicate kidney
failure/end-stage kidney disease?

A. > 90

B. 45-59

C. 15-29

D. < 15
4. 35 year old female comes into the clinic stating that she has painful urination that
occurs often and feels like if she doesn’t go right away she might “pee in her pants”.
Patient also describes pain in her bladder area just above the pubic bone, and has
noticed some redness to her urine. She denies any fever. Negative Lloyd's punch on
physical exam. BUN and creatinine are normal. What is the most likely diagnosis?
A. Intrinsic AKI

B. Pyelonephritis

C. Cystitis

D. Post-renal AKI
4. 35 year old female comes into the clinic stating that she has painful urination that
occurs often and feels like if she doesn’t go right away she might “pee in her pants”.
Patient also describes pain in her bladder area just above the pubic bone, and has
noticed some redness to her urine. She denies any fever. Negative Lloyd's punch on
physical exam. BUN and creatinine are normal. What is the most likely diagnosis?
A. Intrinsic AKI - hematuria would point you to think, but with all of the other symptoms going on would
point you to the kidney

B. Pyelonephritis - fever and flank pain (+ Lloyd punch) would be present

C. Cystitis - dysuria, frequency, urgency, suprapubic pain, and hematuria without fever or flank pain

D. Post-renal AKI - AKI typically shows high levels of BUN and Cr


5. 45 year old male comes in with painful urination with urinary frequency and urgency.
Patient states that they have had a fever with some nausea and vomiting. On physical
exam a Lloyd’s punch is positive. You suspect pyelonephritis. Where would the
associated Chapman’s point be located?

A. 1 inch lateral and 1 inch superior to umbilicus

B. Transverse process of L2

C. 1 inch lateral and 2 inches superior to umbilicus

D. Superior surface of pubic bone near symphysis


5. 45 year old male comes in with painful urination with urinary frequency and urgency.
Patient states that they have had a fever with some nausea and vomiting. On physical
exam a Lloyd’s punch is positive. You suspect pyelonephritis. Where would the
associated Chapman’s point be located?

A. 1 inch lateral and 1 inch superior to umbilicus - anterior, kidneys

B. Transverse process of L2 - posterior, bladder

C. 1 inch lateral and 2 inches superior to umbilicus - anterior, adrenal glands

D. Superior surface of pubic bone near symphysis - anterior, urethra


You will do great! Email me if you have any
questions:

Sam Heywood

samuel.heywood@kansascity.edu
CPR Imaging

Natasha Subbarao
Sahaam Mirza
1. A 57 year old male with a history or hypertension, hyperlipidemia, and diabetes
presents to the emergency room for chest pain that started 5 hours ago. You manage
to get him stabilized but you suspect coronary artery disease. Which imaging modality
is the gold standard for diagnosing this?

a. Coronary Angiography
b. Echocardiogram
c. MUGA Scan
d. X-ray
1. A 57 year old male with a history or hypertension, hyperlipidemia, and diabetes
presents to the emergency room for chest pain that started 5 hours ago. You manage
to get him stabilized but you suspect coronary artery disease. Which imaging modality
is the gold standard for diagnosing this?

a. Coronary Angiography
b. Echocardiogram
c. MUGA Scan
d. X-ray
2. An 89 year old male patient presents to your clinic with his wife who tells you the patient
has been feeling fatigued lately and having increased pitting edema of his lower
extremities. As you look over his previous medical history, you suspect heart failure as one of
your differential diagnoses. Which imaging is appropriate to confirm this suspicion by
measuring ejection fraction specifically?
a. X-Ray
b. CT Angiography
c. Multiple Gated Acquisition Scan
d. Doppler Echo
2. An 89 year old male patient presents to your clinic with his wife who tells you the patient
has been feeling fatigued lately and having increased pitting edema of his lower
extremities. As you look over his previous medical history, you suspect heart failure as one of
your differential diagnoses. Which imaging is appropriate to confirm this suspicion by
measuring ejection fraction specifically?
a. X-Ray
b. CT Angiography
c. Multiple Gated Acquisition Scan
d. Doppler Echo
3. A tall, thin 16 year old female patient with history of Marfan’s presents to your
emergency room with 12/10 tearing chest pain. In order to rule out aortic dissection in
this patient, you should send them to get a:

a. Ultrasound of the aorta


b. CT Angiography of the Aorta
c. Chest X-Ray
d. Color Flow Doppler
3. A tall, thin 16 year old female patient with history of Marfan’s presents to your
emergency room with 12/10 tearing chest pain. In order to rule out aortic dissection in
this patient, you should send them to get a:

a. Ultrasound of the aorta


b. CT Angiography of the Aorta
c. Chest X-Ray
d. Color Flow Doppler
4. A new 3 month old female patient presents to your office. On heart exam you
hear a harsh pansystolic murmur. What imaging modality will help you visualize her
ventricular septal defect?

a. Color Flow Doppler


b. CT without contrast
c. MUGA Scan
d. Doppler Echo
4. A new 3 month old female patient presents to your office. On heart exam you
hear a harsh pansystolic murmur. What imaging modality will help you visualize her
ventricular septal defect?

a. Color Flow Doppler


b. CT without contrast
c. MUGA Scan
d. Doppler Echo
5. A 75 year old man with a history of HTN and valvular disease presents to the
cardiologist. In order to visualize his cardiomegaly, which imaging should be used?

a. Anterior-Posterior Film X-ray


b. Posterior-Anterior Film X-ray
c. CT Angiography
d. Echo
5. A 75 year old man with a history of HTN and valvular disease presents to the
cardiologist. In order to visualize his cardiomegaly, which imaging should be used?

a. Anterior-Posterior Film X-ray


b. Posterior-Anterior Film X-ray
c. CT Angiography
d. Echo
Good luck on the exam!! Feel free to email me
with any questions at
natasha.subbarao@kansascity.edu :)
1. A pregnant women comes into the clinic with a renal problem. You want to complete
imaging of her flank regions. Which is the best modality, given the fact that she is
pregnant?
a. X-Ray
b. MUGA Scan
c. CT Angiography
d. Ultrasound
1. A pregnant women comes into the clinic with a renal problem. You want to complete
imaging of her flank regions. Which is the best modality, given the fact that she is
pregnant?
a. X-Ray
b. MUGA Scan
c. CT Angiography
d. Ultrasound -> There is no radiation risk and does not require contrast (bad for kidneys)
2. Which transducer position is number 3 and what position is the patient in?

4
2

3
2. Which transducer position is number 3 and what position is the patient in?

Apical “Window”
in the Left Lateral
Decubitus
Position

Know these
positions!
3. 65 year old man with a 40 ppd comes in to the clinic and is stating that he has chest
pain that is radiating to the back. You notice on the general appearance exam that he is
tall and skinny. After completing your physical exam, you see that there is unequal BP in
the arms. What imaging should you order?

a. CXR
b. CTA
c. MRI
d. Ultrasound
3. 65 year old man with a 40 ppd comes in to the clinic and is stating that he has chest
pain that is radiating to the back. You notice on the general appearance exam that he is
tall and skinny. After completing your physical exam, you see that there is unequal BP in
the arms. What imaging should you order?

a. CXR
b. CTA -> choice of imaging for Aortic Dissection
c. MRI
d. Ultrasound
4. A 60 year old woman comes into clinic with chest pain. Past medical history include
HTN and valvular disease. Your resident thinks it's best to order an AP view X-ray and
notices that there is a heart enlargement. What problem do you see?

a. AP view can show false enlargement of the heart


b. X-Ray is a contradiction in this patient due to valvular disease
c. X-Ray is not sufficient to diagnose cardiomegaly
d. There was no need for any imaging
4. A 60 year old woman comes into clinic with chest pain. Past medical history include
HTN and valvular disease. Your resident thinks it's best to order an AP view X-ray and
notices that there is a heart enlargement. What problem do you see?

a. AP view can show false enlargement of the heart -> You need to check in the PA view to
diagnose cardiomegaly
b. X-Ray is a contradiction in this patient due to valvular disease
c. X-Ray is not sufficient to diagnose cardiomegaly
d. There was no need for any imaging
5. Rank these imaging modalities in order of ionizing radiation strength

a. Mammogram
b. Chest X-ray
c. CT Scan of the Abdomen and Pelvis
d. Dental X-Ray
e. Average Yearly Environment Exposure
f. Virtual (CT) Colonography
5. Rank these imaging modalities in order of ionizing radiation strength

a. Dental X-Ray
b. Chest X-Ray
c. Mammogram
d. Average Yearly Environment Exposure
e. Virtual (CT) Colonography
f. CT Scan of the Abdomen and Pelvis
Good luck on the exam!! Email me if you have
questions!

sahaam.mirza@kansascity.edu
Lab Questions
Approach to ENT Complaint
LAB
Elizabeth Straus

Andrea Picciotto
A 6 year old boy is brought to your office. His mother reports he
has been more irritable lately and is pulling on his right ear more
than usual. What is the best way to view the eardrum in this
patient?

a. Pull the ear down, out and posterior


b. Pull the ear up, out and posterior
c. Pull the ear up, out and anterior
d. Pull the ear down, out and anterior
A 6 year old boy is brought to your office. His mother reports he
has been more irritable lately and is pulling on his right ear more
than usual. What is the best way to view the eardrum in this
patient?

a. Pull the ear down, out and posterior- this is very clinically relevant
b. Pull the ear up, out and posterior- how you would view an adult’s eardrum
c. Pull the ear up, out and anterior
d. Pull the ear down, out and anterior
Which of the following characteristics would NOT be considered
a normal finding?
a. Pea-sized
b. Mobile
c. Non tender
d. Hard
e. Round
Which of the following characteristics would NOT be considered
a normal finding?
a. Pea-sized
b. Mobile
c. Non tender
d. Hard- make sure you know the abnormal and the normal. Some other abnormal findings would
include tenderness, large, irregular shape, and fixed.
e. Round
A 64 year old woman presents to your office complaining that she
can no longer hear her husband call her from the other room. In
order to evaluate her self reported hearing loss, you perform the
whisper test. What would a normal test result be?

a. Patient correctly repeats a sequence of three numbers or letters


b. Patient correctly identifies which ear is being tested
c. Patient can identify 3 letters or numbers out of 6
d. The test lateralizes to neither ear
A 64 year old woman presents to your office complaining that she
can no longer hear her husband call her from the other room. In
order to evaluate her self reported hearing loss, you perform the
whisper test. What would a normal test result be?
a. Patient correctly repeats a sequence of three numbers or letters
b. Patient correctly identifies which ear is being tested
c. Patient can identify 3 letters or numbers out of 6
d. The test lateralizes to neither ear
The Dix-Hallpike Maneuver is used in helping diagnosis which of
the following?
a. Meniere Disease
b. Benign Paroxysmal Positional Vertigo
c. Labyrinthitis
d. Orthostatic hypertension
The Dix-Hallpike Maneuver is used in helping diagnosis which of
the following?
a. Meniere Disease
b. Benign Paroxysmal Positional Vertigo
c. Labyrinthitis
d. Orthostatic hypertension
A 19 year old nonbinary college student presents to the
students wellness clinic complaining of feeling “icky” the past
few days. They report feeling a fullness in their head, clear
sinus drainage, and fatigue. What Chapman’s points would
you expect to find in this patient?

a. Superior surface of the second rib, 2-3 inches from the sternum
b. C2 at the lamina
c. Superior edge of the proximal clavicle at the medial ⅓
d. On the anterior lateral 2nd rib cartilage
A 19 year old nonbinary college student presents to the
students wellness clinic complaining of feeling “icky” the past
few days. They report feeling a fullness in their head, clear
sinus drainage, and fatigue. What Chapman’s points would
you expect to find in this patient?
a. Superior surface of the second rib, 2-3 inches from the sternum - larynx
b. C2 at the lamina- larynx
c. Superior edge of the proximal clavicle at the medial ⅓- sinuses
d. On the anterior lateral 2nd rib cartilage- tongue

Start memorizing as many of these as you can now! They never go away!
Email me if you have any questions! elizabeth.straus@kansascity.edu
A 32 yr old woman comes in to your clinic with a
complaint of dysphagia. She has a history of T2DM, PTSD,
and depression. Before your ENT exam of the thyroid you
preface her and say “I will be….

a. “Standing behind you as I palpate your thyroid and then I will ask you to swallow for me”
b. “Standing behind as I palpate your thyroid. When you feel ready, swallow. Let me know if you
would like me to stop.”
c. “Standing in front and on the side and will put my hands like this (demonstrate on yourself) to
palpate your thyroid. When you feel ready, swallow. Let me know if you would like me to stop.”
d. “Standing in front of you and on the side to palpate your thyroid. This will be quick”
A 32 yr old patient comes in to your clinic with a
complaint of dysphagia. They have a history of T2DM,
PTSD, and depression. Before your ENT exam of the
thyroid you preface them and say “I will be….
a. “Standing behind you as I palpate your thyroid and then I will ask you to swallow for me”. The posterior approach
can simulate choking. Always be aware of how your your position and the language could affect your patient’s
comfort level.
b. “Standing behind as I palpate your thyroid. When you feel ready, swallow. Let me know if you would like me to
stop.”
c. “Standing in front and on the side and will put my hands like this (demonstrate on yourself) to palpate your
thyroid. When you feel ready, swallow. Let me know if you would like me to stop.” This could be a very
triggering exam for many patients, especially for those who have PTSD. The more information and clarity you
give them before, the better!
d. “Standing in front of you and on the side to palpate your thyroid. This will be quick”
A 32 yr old patient comes in to your clinic with a complaint
of dysphagia. They have a history of T2DM, PTSD, and
depression. During you ENT exam, where do you palpate
the thyroid?
a. Above the cricoid cartilage, at the thyroid isthmus overlying the sternohyoid muscle
b. Below the cricoid cartilage, at the thyroid isthmus, overlying the 1st, 2nd, 3rd tracheal rings
c. Below the cricoid cartilage, at the thyroid isthmus, overlying the 2nd, 3rd, 4th, tracheal rings
d. Above the cricoid cartilage, at the thyroid isthmus, at the cross-section of the cricopharyngeal
muscle and sternohyoid muscle.
A 32 yr old patient comes in to your clinic with a complaint
of dysphagia. They have a history of T2DM, PTSD, and
depression. During you ENT exam, where do you palpate
the thyroid?
a. Above the cricoid cartilage, at the thyroid isthmus overlying the sternohyoid muscle
b. Below the cricoid cartilage, at the thyroid isthmus, overlying the 1st, 2nd, 3rd tracheal rings
c. Below the cricoid cartilage, at the thyroid isthmus, overlying the 2nd, 3rd, 4th, tracheal rings
d. Above the cricoid cartilage, at the thyroid isthmus, at the cross-section of the cricopharyngeal
muscle and sternohyoid muscle.
76 yo woman comes in with a complaint of not being
able to hear her family talk in a restaurant. You
perform the Whisper Test by….
a. Whispering three letters behind the patient. It is abnormal if the patient incorrectly identifies ⅔
letters.
b. Placing two fingers a couple of inches away from one ear and rubbing together. Patient
whispers what side he heard it on
c. Whispering a 3 series of numbers and letters. It is abnormal if the patient incorrectly identifies
4 of the 6
d. Whispering a 3 series of numbers and letters on one side of the ear. It is a sensorineural hearing
loss if the patient can’t hear on one side.
76 yo woman comes in with a complaint of not being
able to hear her family talk in a restaurant. You
perform the Whisper Test by….
a. Whispering three letters behind the patient. It is abnormal if the patient incorrectly identifies ⅔
letters.
b. Placing two fingers a couple of inches away from one ear and rubbing together. Patient
whispers what side he heard it on
c. Whispering a 3 series of numbers and letters. It is abnormal if the patient incorrectly
identifies 4 of the 6.
d. Whispering a 3 series of numbers and letters on one side of the ear. It is a sensorineural hearing
loss if the patient can’t hear on one side.
An 18 yo old with a recent sinus infection come to
you with a complaint of congestion in his face. You
suggest using the Galbreath Technique. Your hand
placement for this technique would be…
a. Using caudad hand, place 3rd and 4th fingers in front of and behind ear, respectively
b. Place fingertips below the inferior rim of the mandible
c. Place fingers of caudal hand along posterior ramus of mandible, place hypothenar eminence
along body of mandible
d. Using caudal hand, place palmar aspect of fingers over the SCM, near the angle of of the
mandible
An 18 yo old with a recent sinus infection come to
you with a complaint of congestion in his face. You
suggest using the Galbreath Technique. Your hand
placement for this technique would be…
a. Using caudad hand, place 3rd and 4th fingers in front of and behind ear, respectively
Auricular Drainage
b. Place fingertips below the inferior rim of the mandible Submandibular Drainage
c. Place fingers of caudal hand along posterior ramus of mandible, place hypothenar
eminence along body of mandible Mandibular Drainage = Galbreath Technique
d. Using caudal hand, place palmar aspect of fingers over the SCM, near the angle of of the
mandible Cervical Chain Drainage
What is the tonsil chapman point?

a. The superior surface of the second rib, 2-3 inches from the sternum
b. On the anterior lateral 2nd rib cartilage
c. The superior edge of the proximal clavicle at the medial ⅓
d. Between the 1st and 2nd ribs just lateral to the manubrium
What is the tonsil chapman point?

a. The superior surface of the second rib, 2-3 inches from the sternum Larynx
b. On the anterior lateral 2nd rib cartilage Tongue
c. The superior edge of the proximal clavicle at the medial ⅓ Sinuses
d. Between the 1st and 2nd ribs just lateral to the manubrium Tonsils
If you have any questions, feel free to email me!

andrea.picciotto@kansascity.edu

Goodluck!
Approach to Endocrine
Complaint & Autonomics OMM
Lab
Lamia Faruk and Marenda Biggs
1. A 45 year old male comes to the ER presenting with severe right flank pain. The patient is writhing in pain on the bed, and when
you ask him more about his symptoms, he says that the pain “goes in and out” alot. You take a urine sample, which is positive for
+3 hemoglobin. You perform a non-contrast CT and diagnose him with nephrolithiasis. Where would you expect to find a posterior
Chapman’s point?
A. About 1 inch lateral to and inch above the umbilicus
B. Superior transverse process of L2
C. Lateral portion of the right T11-12 intertransverse space.
D. Superior portion of right T12 near the L1 facet joint.
1. A 45 year old male comes to the ER presenting with severe right flank pain. The patient is writhing in pain on the bed, and when
you ask him more about his symptoms, he says that the pain “goes in and out” alot. You take a urine sample, which is positive for
+3 hemoglobin. You perform a non-contrast CT and diagnose him with nephrolithiasis. Where would you expect to find a
posterior Chapman’s point?
A. About 1 inch lateral to and inch above the umbilicus
B. Superior transverse process of L2
C. Lateral portion of the right T11-12 intertransverse space.
D. Superior portion of right T12 near the L1 facet joint.
2. A 32 year old female arrives at your clinic complaining of not being able to catch her breath and feeling dizzy randomly
throughout the day. You see her eyes slightly bulging and you perform a cardiac exam on her, noting that her heart rate is fast.
Upon palpating her thyroid, you think she may have Grave’s Disease. What would you expect her thyroid to feel like?
A. Enlarged, Firm, and Nodular
B. Small and Fluctuant
C. Diffusely Enlarged
D. Fibrotic
2. A 32 year old female arrives at your clinic complaining of not being able to catch her breath and feeling dizzy randomly
throughout the day. You see her eyes slightly bulging and you perform a cardiac exam on her, noting that her heart rate is
fast. Upon palpating her thyroid, you think she may have Grave’s Disease. What would you expect her thyroid to feel like?
A. Enlarged, Firm, and Nodular
B. Small and Fluctuant
C. Diffusely Enlarged
D. Fibrotic
3. A 55 year old obese male with a past medical history of hyperlipidemia and hypertension presents to your office for an annual
checkup. He tells you that he’s been recently worried about his health because he has been experiencing some new onset
chest pain that seems to get worse whenever he’s doing his daily mile walk. He does state that resting makes him feel better.
During your physical examination, you perform an osteopathic exam, and look for any somatic dysfunctions. What diagnosis
would you expect in this case?

A. N T7-T9 RLSL
B. E T4-T6 RRSR
C. N T5-T8 RLSR
D. N T4-T6 RLSL
3. A 55 year old obese male with a past medical history of hyperlipidemia and hypertension presents to your office for
an annual checkup. He tells you that he’s been recently worried about his health because he has been experiencing
some new onset chest pain that seems to get worse whenever he’s doing his daily mile walk. He does state that
resting makes him feel better. During your physical examination, you perform an osteopathic exam, and look for any
somatic dysfunctions. What diagnosis would you expect in this case?

A. N T7-T9 RLSL
B. E T4-T6 RRSR
C. N T5-T8 RLSR
D. N T4-T6 RLSL
4. A 26 year old female arrives to your outpatient clinic with her four year old daughter who has had a sore throat and a fever for
the past week. You examine the daughter’s throat, and you see a gray white exudate at her pharynx. Her mother denies that
her daughter has had a cough during this time. During your physical exam, you perform an osteopathic examination. What
would you expect a Chapman’s point in the 1st intercostal space (near the sternum) to feel like?

A. Large and soft nodule that is tender


B. A fluctuant and small nodule that is tender
C. A round nodule that is somewhat firm and has radiating pain when palpated
D. A smooth and firm nodule that has no pain when palpated
4. A 26 year old female arrives to your outpatient clinic with her four year old daughter who has had a sore throat and
a fever for the past week. You examine the daughter’s throat, and you see a gray white exudate at her pharynx. Her
mother denies that her daughter has had a cough during this time. During your physical exam, you perform an
osteopathic examination. What would you expect a Chapman’s point in the 1st intercostal space (near the sternum)
to feel like?

A. Large and soft nodule that is tender


B. A fluctuant and small nodule that is tender
C. A round nodule that is somewhat firm and has radiating pain when palpated
D. A smooth and firm nodule that has no pain when palpated
5. A 25 year old female arrives at your family medicine clinic with her six year old daughter who has had a stomach bug for the past day
or so. The mother reports that her daughter has been having diarrhea multiple times throughout the day, and asks if there is anything
you can do to help. You decide to perform a soft tissue technique that involves the sacrum. How will you use respiratory motion in your
technique?
A. Resist extension of the sacrum upon inhalation and resist flexion of the sacrum during exhalation
B. Accentuate extension of the sacrum upon inhalation and accentuate flexion of the sacrum during exhalation
C. Resist flexion of the sacrum upon inhalation and resist extension of the sacrum during exhalation.
D. Accentuate flexion of the sacrum upon inhalation and accentuate extension of the sacrum during exhalation
5. A 25 year old female arrives at your family medicine clinic with her six year old daughter who has had a stomach bug for the past day
or so. The mother reports that her daughter has been having diarrhea multiple times throughout the day, and asks if there is anything
you can do to help. You decide to perform a soft tissue technique that involves the sacrum. How will you use respiratory motion in your
technique?
A. Resist extension of the sacrum upon inhalation and resist flexion of the sacrum during exhalation
B. Accentuate extension of the sacrum upon inhalation and accentuate flexion of the sacrum during exhalation
C. Resist flexion of the sacrum upon inhalation and resist extension of the sacrum during exhalation.
D. Accentuate flexion of the sacrum upon inhalation and accentuate extension of the sacrum during exhalation
GOOD LUCK ON YOUR MIDTERM Y’ALL!
If you have any questions, feel free to email me at farhana.faruk@kansascity.edu!!
Attached is a youtube video that I used to learn some of the Chapman’s Points. If any of these do not
align with KCU, always go with what our professors say first, but it makes it a little more enjoyable to
learn.
https://www.youtube.com/watch?v=Lq7zsi60tVc&t=212s
https://www.youtube.com/watch?v=xXOppdQ_HFk
A patient presents with complaints of cough and shortness of breath. Because the patient is weak, you choose to treat with rib raising.
Please select the correct statement about rib raising:
A. Hands are aligned along the transverse processes of the corresponding vertebrae
B. The goal of rib raising is sympathetic normalization
C. Physician stands on the opposite side of the ribs they are treating
D. The goal of rib raising is parasympathetic normalization
A patient presents with complaints of cough and shortness of breath. Because the patient is weak, you choose to treat with rib raising.
Please select the correct statement about rib raising:
A. Hands are aligned along the transverse processes of the corresponding vertebrae
B. The goal of rib raising is sympathetic normalization
C. Physician stands on the opposite side of the ribs they are treating
D. The goal of rib raising is parasympathetic normalization
32-year old female presents to the clinic with complains of “feeling cold all the time.” Upon further examination, you note her hair and
skin are both very dry. You get lab work done and make the diagnosis of hypothyroidism. What is another expected finding in this
patient?
A. Tremor
B. Tachycardia
C. Hyporeflexia
D. Proptosis
32-year old female presents to the clinic with complains of “feeling cold all the time.” Upon further examination, you note her hair and
skin are both very dry. You get lab work done and make the diagnosis of hypothyroidism. What is another expected finding in this
patient?
A. Tremor
B. Tachycardia
C. Hyporeflexia
D. Proptosis
While working up a patient with newly diagnosed diabetes mellitus type 2, you tell them it is important for them to make sure they are
monitoring their blood sugar and look at a certain part of their body more closely than before they had diabetes. You also note to
yourself to look more closely at this body part during upcoming check ups. What is this body part?
A. Scalp
B. Feet
C. Eye
D. All the above
E. A and B
F. B and C
While working up a patient with newly diagnosed diabetes mellitus type 2, you tell them it is important for them to make sure they are
monitoring their blood sugar and look at a certain part of their body more closely than before they had diabetes. You also note to
yourself to look more closely at this body part during upcoming check ups. What is this body part?
A. Scalp
B. Feet
C. Eye
D. All the above
E. A and B
F. B and C
During an endocrine-specific physical exam, you perform a test that includes inflating a cuff over the patient’s arm for 3 minutes. During
this exam, you note wrist flexor muscle spasms. What is the most likely finding on this patient’s blood work?
A. Hyponatremia
B. Hypernatremia
C. Hypocalcemia
D. Hypercalcemia
E. Hypokalemia
F. Hyperkalemia
During an endocrine-specific physical exam, you perform a test that includes inflating a cuff over the patient’s arm for 3 minutes. During
this exam, you note wrist flexor muscle spasms. What is the most likely finding on this patient’s blood work?
A. Hyponatremia
B. Hypernatremia
C. Hypocalcemia
D. Hypercalcemia
E. Hypokalemia
F. Hyperkalemia
Patient presents with complaints of “these weird colored stretch marks on my stomach.” Further examination reveals a large abdomen
and round face with thin extremities. What is a common cause of this condition?
A. NSAID excess
B. Cortisol excess
C. Catecholamine excess
D. Mineralocorticoid excess
Patient presents with complaints of “these weird colored stretch marks on my stomach.” Further examination reveals a large abdomen
and round face with thin extremities. What is a common cause of this condition?
A. NSAID excess
B. Cortisol excess
C. Catecholamine excess
D. Mineralocorticoid excess
Y’all are doing great! Best of
luck and email if you have any
questions!
Marenda Biggs
marenda.biggs@kansascity.edu
Approach to UE/LE
Counterstrain Lab
Joanna Yuen & Minah Tariq
1. Tom is a professional baseball player and is the pitcher for Kansas City Royals.
He’s been experiencing pain in the right shoulder that sometimes radiates to the
side of his arm. Empty can test is positive on exam. What positioning would be
most effective at treating this injury?

a) F Add ER
b) E Abd IR
c) E Add IR
d) F Abd ER
1. Tom is a professional baseball player and is the pitcher for Kansas City Royals.
He’s been experiencing pain in the right shoulder that sometimes radiates to the
side of his arm. Empty can test is positive on exam. What positioning would be
most effective at treating this injury?

a) F Add ER
b) E Abd IR
c) E Add IR
d) F Abd ER
2. Rebecca has been under the weather and has not been able to stomach anything but soup.
She heats up a can of chicken noodle soup and pours it into a bowl. While attempting to eat
in bed, she notices she can’t hold the bowl of soup in her palms without significant pain. In
clinic, you decide to treat her with counterstrain with extension, supination, and valgus.
What tenderpoint are you treating?
a) Medial epicondyle
b) Radial head (lateral)
c) Extensor carpi radialis
d) Lateral epicondyle
2. Rebecca has been under the weather and has not been able to stomach anything but soup.
She heats up a can of chicken noodle soup and pours it into a bowl. While attempting to eat
in bed, she notices she can’t hold the bowl of soup in her palms without significant pain. In
clinic, you decide to treat her with counterstrain with extension, supination, and valgus.
What tenderpoint are you treating?
a) Medial epicondyle
b) Radial head (lateral)
c) Extensor carpi radialis
d) Lateral epicondyle
3. You get assigned counterstrain for your technique practical. Verbalize and list the steps in
the correct order in order to get full points:
a) Reassess TART
b) Return to neutral
c) Establish a tenderness scale
d) Place the pt into the position of ease until at least 70% relief achieved
e) Monitor the tender point
f) Find the greatest tenderpoint
g) Hold for at least 90 seconds
3. You get assigned counterstrain for your technique practical. Verbalize and list the steps in the
correct order in order to get full points:
a) Reassess TART
b) Return to neutral
c) Establish a tenderness scale
d) Place the pt into the position of ease until at least 70% relief achieved
e) Monitor the tender point
f) Find the greatest tenderpoint
g) Hold for at least 90 seconds

F, C, E, D, G, B, A
4. 22 y/o cis-gender male presents to clinic with pain in the lateral hamstring. He was
doing Romanian deadlifts at the gym yesterday and went a little too hard. You decide
to treat the patient’s pain with counterstrain, what is the location of this tenderpoint?
a) Just inferior to iliac crest in body of TFL
b) Along the iliotibial band, distal to the greater trochanter
c) Distal aspect of biceps femoris, near attachment to the posterolateral surface of
the fibular head
d) Distal aspect of the medial hamstring tendons near their attachment to the
posterior medial surface of the tibial condyle
4. 22 y/o cis-gender male presents to clinic with pain in the lateral hamstring. He was
doing Romanian deadlifts at the gym yesterday and went a little too hard. You decide
to treat the patient’s pain with counterstrain, what is the location of this tenderpoint?
a) Just inferior to iliac crest in body of TFL
b) Along the iliotibial band, distal to the greater trochanter
c) Distal aspect of biceps femoris, near attachment to the posterolateral surface
of the fibular head
d) Distal aspect of the medial hamstring tendons near their attachment to the
posterior medial surface of the tibial condyle
5. Pt is a 19 y/o transgender male, AFAB, who is a professional dancer presenting with right knee pain
for the last 6 weeks. Pt states he was coming down from a jump in the air when he felt a sudden twist
in his right knee. McMurray test is positive. Pt has had some improvement with PT, but you would
like to treat the pt with counterstrain to encourage the healing process. Which of the following
treatment positions can be used for this technique?
a) F Abd ir/er
b) fABD
c) F IR Add
d) F IR
e) a and c
5. Pt is a 19 y/o transgender male, AFAB, who is a professional dancer presenting with right knee pain for the
last 6 weeks. Pt states he was coming down from a jump in the air when he felt a sudden twist in his right knee.
McMurray test is positive. Pt has had some improvement with PT, but you would like to treat the pt with
counterstrain to encourage the healing process. Which of the following treatment positions can be used for
this technique?
a) F Abd ir/er
b) fABD
c) F IR Add
d) F IR
e) a and c
Positive McMurray likely indicated a meniscal injury. Exact location (medial v lateral) isn’t specified. As of
now, A (lateral) and C (medial) are possible positions you can use to treat a meniscal tenderpoint.
Good Luck!
Reach out if you have any questions.
Joanna Yuen
joanna.yuen@kansascity.edu
1. A 25 year old pt comes into your clinic saying that they hurt their right ankle playing basketball. They were running
and slipped on the court, when their foot “twisted inwards” and they point to the outside of their ankle. What is the
correct location of the tender point to treat this patient’s somatic dysfunction with counterstrain?
a. Anterior and inferior to the medial malleolus in the sinus tarsi
b. Anterior and superior to the lateral malleolus in the sinus tarsi
c. Anterior and inferior to the lateral malleolus in the sinus tarsi
d. Posterior and inferior to the lateral malleolus in the sinus tarsi
1. A 25 year old pt comes into your clinic saying that they hurt their right ankle playing basketball. They were running
and slipped on the court, when their foot “twisted inwards” and they point to the outside of their ankle. What is the
correct location of the tender point to treat this patient’s somatic dysfunction with counterstrain?
a. Anterior and inferior to the medial malleolus in the sinus tarsi
b. Anterior and superior to the lateral malleolus in the sinus tarsi
c. Anterior and inferior to the lateral malleolus in the sinus tarsi
d. Posterior and inferior to the lateral malleolus in the sinus tarsi
2. The previous patient has a very common sports-related injury and sometimes the patients are able to withstand
direct treatments to the injury, but of course if they do not consent to a direct treatment as it could be too
uncomfortable, counterstrain is still an excellent indirect treatment option that can provide relief and functionality. For
the same 25 year old patient, correctly take them to their indirect barrier to treat this injury.
a. Apply eversion force to foot and ankle with slight external rotation until the tenderness is ≥ 70% reduction
b. Apply inversion force to foot and ankle with slight external rotation until the tenderness is ≥ 70% reduction
c. Dorsiflex the foot and apply eversion force to foot and ankle with slight external rotation until the tenderness
is ≥ 70% reduction
d. Plantarflex the foot and apply inversion force to foot and ankle slight internal rotation until the tenderness is ≥
70% reduction
2. The previous patient has a very common sports-related injury and sometimes the patients are able to withstand
direct treatments to the injury, but of course if they do not consent to a direct treatment as it could be too
uncomfortable, counterstrain is still an excellent indirect treatment option that can provide relief and functionality. For
the same 25 year old patient, correctly take them to their indirect barrier to treat this injury.
a. Apply eversion force to foot and ankle with slight external rotation until the tenderness is ≥ 70%
reduction
b. Apply inversion force to foot and ankle with slight external rotation until the tenderness is ≥ 70% reduction
c. Dorsiflex the foot and apply eversion force to foot and ankle with slight external rotation until the tenderness
is ≥ 70% reduction
d. Plantarflex the foot and apply inversion force to foot and ankle slight internal rotation until the tenderness is ≥
70% reduction
3. You are in residency and meet a budding 3rd year osteopathic medical student. They are eager to learn and try to
show off their OMM skills in front of you so they ask to perform counterstrain on your patient (with the patient’s
consent) for their ankle injury. They begin by locating the tibialis anterior tender point, then they move the patient into
inversion until the patient does not report any pain when the tender point is palpated. They then hold the position for
70 seconds and move the patient back into a neutral position, and look to you proudly thinking they’ve REALLY
showed off. You sadly tell them the 3 mistakes they made performing the treatment, which are…
a. They did not establish a pain scale for the patient
b. They did not hold the treatment position for 90 seconds
c. They did not reassess the tender point after the treatment to ensure it was successful
d. All of the above
e. A and c
3. You are in residency and meet a budding 3rd year osteopathic medical student. They are eager to learn and try to
show off their OMM skills in front of you so they ask to perform counterstrain on your patient (with the patient’s
consent) for their ankle injury. They begin by locating the tibialis anterior tender point, then they move the patient into
inversion until the patient does not report any pain when the tender point is palpated. They then hold the position for
70 seconds and move the patient back into a neutral position, and look to you proudly thinking they’ve REALLY
showed off. You sadly tell them the 3 mistakes they made performing the treatment, which are…
a. They did not establish a pain scale for the patient
b. They did not hold the treatment position for 90 seconds
c. They did not reassess the tender point after the treatment to ensure it was successful
d. All of the above
e. A and c

Knowing the correct steps for counterstrain is crucial! You must be able to not only list them in
correct order but should also recognize when a step is missing.
4. A 40 year old male presents to your very popular clinic complaining of pain “in the middle of his foot”. He’s been a
runner for about 10 years and says he’s previously has HVLA done on the same injury and it felt great to hear that
crack when the physician finished the treatment, but the foot still feels sore soon after so he wants to try something
else maybe more gentle. Recommending counterstrain as an indirect technique, you first palpate his foot and feel 2
distinct tender points! The first one is on the anterior aspect of the plantar surface of the calcaneus at the attachment
of the plantar fascia. What muscle/tendon would you be treating at this tender point?
a. Abductor digiti minimi
b. Flexor digitorum brevis
c. Flexor hallucis longus tendon
d. Quadratus plantae
4. A 40 year old male presents to your very popular clinic complaining of pain “in the middle of his foot”. He’s been a
runner for about 10 years and says he’s previously has HVLA done on the same injury and it felt great to hear that
crack when the physician finished the treatment, but the foot still feels sore soon after so he wants to try something
else maybe more gentle. Recommending counterstrain as an indirect technique, you first palpate his foot and feel 2
distinct tender points! The first one is on the anterior aspect of the plantar surface of the calcaneus at the attachment
of the plantar fascia. What muscle/tendon would you be treating at this tender point?
a. Abductor digiti minimi
b. Flexor digitorum brevis
c. Flexor hallucis longus tendon
d. Quadratus plantae
5. Excellent! You’ve successfully treated the patient’s first somatic dysfunction and he says the treatment felt great and
the tender point was at a 0/10 for tenderness. Now onto the second somatic dysfunction. The patient says he knows
this tender point was on his navicular and he’s had this treated before as well, but only with direct treatments. Using
counterstrain, how would you position your patient to treat this somatic dysfunction?
a. Standing ipsilateral to TP, the patient’s knee is flexed, dorsum of the foot on doc’s thigh, plantar flexion of
subtalar joint, supination of forefoot until the tenderness is ≥ 70% reduction
b. Standing contralateral to TP, the patient’s knee is flexed, dorsum of the foot on doc’s thigh, plantar flexion of
subtalar joint, supination of forefoot until the tenderness is ≥ 70% reduction
c. Standing contralateral to TP, the patient’s knee is flexed, dorsum of the foot on doc’s thigh, plantar flexion of
subtalar joint, pronation of forefoot until the tenderness is ≥ 70% reduction
d. I had no idea the foot could even have so many possible treatments…
5. Excellent! You’ve successfully treated the patient’s first somatic dysfunction and he says the treatment felt great and
the tender point was at a 0/10 for tenderness. Now onto the second somatic dysfunction. The patient says he knows
this tender point was on his navicular and he’s had this treated before as well, but only with direct treatments. Using
counterstrain, how would you position your patient to treat this somatic dysfunction?
a. Standing ipsilateral to TP, the patient’s knee is flexed, dorsum of the foot on doc’s thigh, plantar
flexion of subtalar joint, supination of forefoot until the tenderness is ≥ 70% reduction
b. Standing contralateral to TP, the patient’s knee is flexed, dorsum of the foot on doc’s thigh, plantar flexion of
subtalar joint, supination of forefoot until the tenderness is ≥ 70% reduction
c. Standing contralateral to TP, the patient’s knee is flexed, dorsum of the foot on doc’s thigh, plantar flexion of
subtalar joint, pronation of forefoot until the tenderness is ≥ 70% reduction
d. I had no idea the foot could even have so many possible treatments…
You got this! Go forth and be the ortho/OMM masters we
need.
Contact me if y’all have any questions!
minah.tariq@kansascity.edu

This could
be you…
UE/LE HVLA Lab

By: Spencer Phillips & Lauren Chi


UE/LE HVLA Lab

1. Your friend complains of tenderness around the area of the radial head. As you supinate the wrist,
they report increased pain. Which of the following is true?
a. Supination is coupled with anterior glide
b. Supination is coupled with posterior glide
c. Supination is coupled with with inferior glide
d. Supination is coupled with superior glide
UE/LE HVLA Lab

1. Your friend complains of tenderness around the area of the radial head. As you supinate the wrist,
they report increased pain. Which of the following is true?
a. Supination is coupled with anterior glide
b. Supination is coupled with posterior glide
c. Supination is coupled with with inferior glide
d. Supination is coupled with superior glide
UE/LE HVLA Lab

2. Which of the following is true?

A. Normal wrist flexion is between 60-70 degrees


B. Normal wrist extension is 90-110 degrees
C. Normal ulnar deviation is 30-40 degrees
D. Normal radial deviation is 10-20 degrees
E. All of the following are true
UE/LE HVLA Lab

2. Which of the following is true?

A. Normal wrist flexion is between 60-70 degrees


B. Normal wrist extension is 90-110 degrees
C. Normal ulnar deviation is 30-40 degrees
D. Normal radial deviation is 10-20 degrees
E. All of the following are true
UE/LE HVLA Lab

3. You are rotating in a family practice clinic in Coffeyville, KS. Your attending tells you that your patient is
a relatively healthy 35 year old (they/them/their), though their left wrist has a somatic dysfunction that is
named “Left flexed wrist SD”. Which of the following is true?

A. Treating with HVLA is contraindicated in this patient


B. Treating with HVLA would include providing a whip-like flexion to extension movement
C. Treating with HVLA would include providing a whip-like extension to flexion movement
D. Treating with HVLA would first start by establishing a tenderness scale.
E. Treating with HVLA may be possible, but I want to see my preceptor do it first.
UE/LE HVLA Lab

3. You are rotating in a family practice clinic in Coffeyville, KS. Your attending tells you that your patient is
a relatively healthy 35 year old (they/them/their), though their left wrist has a somatic dysfunction that is
named “Left flexed wrist SD”. Which of the following is true?

A. Treating with HVLA is contraindicated in this patient


B. Treating with HVLA would include providing a whip-like flexion to extension movement
C. Treating with HVLA would include providing a whip-like extension to flexion movement
D. Treating with HVLA would first start by establishing a tenderness scale.
E. Treating with HVLA may be possible, but I want to see my preceptor do it first.
UE/LE HVLA Lab

4. You are rotating in a family practice clinic in Coffeyville, KS for another week and the next day you find
a patient with a fibular head somatic dysfunction. You diagnose a Right Anterior Fibular Head SD, well
done! Now, how do you treat this with HVLA?

A. Treating with HVLA is contraindicated because I am not comfortable doing it.


B. Treating with HVLA requires lateralization of forces via external rotation
C. Treating with HVLA requires lateralization of forces via internal rotation
D. Treating with HVLA requires an anterior thrust
UE/LE HVLA Lab

4. You are rotating in a family practice clinic in Coffeyville, KS for another week and the next day you find
a patient with a fibular head somatic dysfunction. You diagnose a Right Anterior Fibular Head SD, well
done! Now, how do you treat this with HVLA?

A. Treating with HVLA is contraindicated because I am not comfortable doing it.


B. Treating with HVLA requires lateralization of forces via external rotation
C. Treating with HVLA requires lateralization of forces via internal rotation
D. Treating with HVLA requires an anterior thrust
UE/LE HVLA Lab

5. You are with your attending in clinic and she says “Quick! What barrier do you engage in HVLA?” What
is your response to her?

A. Physiologic Barrier
B. Anatomic Barrier
C. Restrictive Barrier
D. Great Barrier Reef
UE/LE HVLA Lab

5. You are with your attending in clinic and she says “Quick! What barrier do you engage in HVLA?” What
is your response to her?

A. Physiologic Barrier
B. Anatomic Barrier
C. Restrictive Barrier
D. Great Barrier Reef
UE/LE HVLA Lab
6. You are working diligently until suddenly, Michael Scott summons you to come to
his office. He said he was wrestling with Dwight (again) and hurt his right arm. As the
awesome OMM student, you begin to palpate his arm and find a right radial head
pronated SD. You are about to knock his socks off with your new trick - UE HVLA.
What is the restrive barrier and what is the activating force for treating his SD?

A. RB = Flex elbow and Pronate; AF = Hyperflexion while simultaneously thrusting radial head posteriorly
B. RB = Extend and Supinate; AF = Hyperextension while simultaneously thrusting radial head anteriorly
C. RB = Supination; AF = pronation
D. RB = Pronation; AF = Supination
UE/LE HVLA Lab
6. You are working diligently until suddenly, Michael Scott summons you to come to
his office. He said he was wrestling with Dwight (again) and hurt his right arm. As the
awesome OMM student, you begin to palpate his arm and find a right radial head
pronated SD. You are about to knock his socks off with your new trick - UE HVLA.
What is the restrive barrier and what is the activating force for treating his SD?

A. RB = Flex elbow and Pronate; AF = Hyperflexion while simultaneously thrusting radial head posteriorly
a. For Anterior Radial Head SD - HVLA
B. RB = Extend and Supinate; AF = Hyperextension while simultaneously thrusting radial head anteriorly
C. RB = Supination; AF = pronation
a. For Posterior Radial Head SD - MET
D. RB = Pronation; AF = Supination
a. For Anterior Radial Head SD - MET
UE/LE HVLA Lab

7. Michael is amazed with your powers and calls the rest of the team over to you! First, we have Pam.
After of course getting her consent to perform OMM, you palpate and find a right wrist SD.

What is the activating force for a right Wrist Extension/Ventral Carpal SD HVLA?

A. Pronate elbow. Grasp patient’s hand, thumbs contacting dorsally at the proximal carpal bones
(radiocarpal joint)
B. Deliver a whip-like thrust moving from extension to flexion through the carpal dysfunction
C. Deliver a whip-like thrust moving from flexion to extension through the carpal dysfunction
D. Exert a rapid hyperextension force while simultaneously thrusting the wrist anteriorly
E. Exert a rapid hyperflexion force while simultaneously thrusting the radial head posteriorly
UE/LE HVLA Lab

7. Michael is amazed with your powers and calls the rest of the team over to you! First, we have Pam. After of coursegetting
her consent to perform OMM, you palpate and find a right wrist SD. What is the activating force for a right Wrist
Extension/Ventral Carpal SD HVLA?

A. Pronate elbow. Grasp patient’s hand, thumbs contacting dorsally at the proximal carpal bones (radiocarpal joint)
a. Set up for both wrist flexion and extension HVLA
B. Deliver a whip-like thrust moving from extension to flexion through the carpal dysfunction
C. Deliver a whip-like thrust moving from flexion to extension through the carpal dysfunction
a. For wrist flexion HVLA

What are these bottom two?

D. Exert a rapid hyperextension force while simultaneously thrusting the wrist anteriorly
E. Exert a rapid hyperflexion force while simultaneously thrusting the radial head posteriorly
Remember, HVLA is a DIRECT technique. So,
you will take the patient to where they like to go
(e.g. SD Wrist extension → take into extension),
and then move patient to their RB (e.g. where
they don’t like to go - flexion)
UE/LE HVLA Lab
8. Jim is right after Pam. You find a left leg SD and have located the tibia
to have restricted ROM. He looks at you smiling and says, “Don’t ask.”
Probably has something to do with plotting another prank on Dwight.

What is the direction of the thrust for an anterior a tibia on femur SD


using HVLA?

A. Deliver posterior thrust with the thumbs


B. Deliver anterior (tug) thrust with the index and long fingers.
C. Supinate the foot
D. Pronate the foot
UE/LE HVLA Lab
8. Jim is right after Pam. You find a left leg SD and have located the tibia
to have restricted ROM. He looks at you smiling and says, “Don’t ask.”
Probably has something to do with plotting another prank on Dwight.

What is the direction of the thrust for an anterior a tibia on femur SD


using HVLA?

A. Deliver posterior thrust with the thumbs


B. Deliver anterior (tug) thrust with the index and long fingers.
C. Supinate the foot
D. Pronate the foot
UE/LE HVLA Lab
9. You are just about to head to lunch with suddenly, you see Creed sitting in your
chair, waiting to be your next patient. You definitely don’t recall hearing him enter, but
proceed onwards. He begins to take off his right sock and shoe and lifts it onto your
desk saying he did something to it. You evaluate Creed’s ankle find the following ROM:
Ankle Inversion 20°, Ankle Eversion 5 , and Talocalcaneal Inversion and Eversion: 5 °.

If Creed had a talotibial SD, how would you treat Creed’s SD using HVLA?

A. Internally rotate distal leg for anterior fib. head


B. Externally rotate distal leg for anterior fib. head
C. Caudad traction with eversion or abduction position of the ankle
D. Caudad traction with inversion or adduction of the ankle
UE/LE HVLA Lab
9. You are just about to head to lunch with suddenly, you see Creed sitting in your chair, waiting to be your next patient.
You definitely don’t recall hearing him enter, but proceed onwards. He begins to take off his right sock and shoe and lifts it
onto your desk saying he did something to it. You evaluate Creed’s ankle find the following ROM: Ankle Inversion 20°,
Ankle Eversion 5 , and Talocalcaneal Inversion and Eversion: 5 °.

If Creed had a talotibial SD, how would you treat Creed’s SD using HVLA?

A. Internally rotate distal leg for anterior fib. head Patient has limited ROM with
B. Externally rotate distal leg for anterior fib. head Ankle Eversion →
C. Caudad traction with eversion or abduction position of the ankle This is an Ankle Inversion SD!
D. Caudad traction with inversion or adduction of the ankle
a. Eversion SD
UE/LE HVLA Lab
10. You are done for the day. Another productive and
interesting day at Dunder Mifflin. You are at your car when
suddenly, Dwight pops out from the bushes to surprise you. In
the process of doing so, he inverted his ankle, spraining it in the
process. You assess for a Posterior fibular head SD.

What is the set up to localize all the barriers for a Posterior


Fibular Head SD?

A. Invert, dorsiflex foot and ankle and internally rotate the leg, while fully flexing the hip and knee
B. Evert, dorsiflex foot and ankle and externally rotate the leg, while fully flexing the hip and knee
C. Internally rotate distal leg for anterior fib. head
D. Externally rotate distal leg for anterior fib. head
UE/LE HVLA Lab
10. You are done for the day. Another productive and
interesting day at Dunder Mifflin. You are at your car when
suddenly, Dwight pops out from the bushes to surprise you. In
the process of doing so, he inverted his ankle, spraining it in the
process. You assess for a Posterior fibular head SD.

What is the set up to localize all the barriers for a Posterior


Fibular Head SD?

A. Invert, dorsiflex foot and ankle and internally rotate the leg, while fully flexing the hip and knee
B. Evert, dorsiflex foot and ankle and externally rotate the leg, while fully flexing the hip and knee
C. Internally rotate distal leg for anterior fib. Head
a. Anterior Fibular head SD
D. Externally rotate distal leg for anterior fib. head
Good luck! You got this!

Lauren Chi (lauren.chi@kansascity.edu)


Approach to Cardiac
Complaint & Rib MET Lab
Pt 1: Parker Howe
Pt 2: Victor Phan
A student doctor is performing a cardiac exam on a 10 year old boy when they hear a late systolic crescendo type
murmur. The murmur is heard best when they were auscultating over the left 5th intercostal space. What is the most
likely cause of the murmur that they heard?

A. Tricuspid valve stenosis


B. Aortic valve regurgitation
C. Pulmonic valve stenosis
D. Mitral valve prolapse
A student doctor is performing a cardiac exam on a 10 year old boy when they hear a late systolic crescendo type
murmur. The murmur is heard best when they were auscultating over the left 5th intercostal space. What is the most
likely cause of the murmur that they heard?

A. Tricuspid valve stenosis


B. Aortic valve regurgitation
C. Pulmonic valve stenosis
D. Mitral valve prolapse

You can answer this question by


knowing the location at which
each valve murmur will be heard
the clearest, without knowing yet
the exact specifics of how
murmurs typically sound
A physician is working with a scribe while they are performing a cardiac exam on a 55 year old male. The patient is
presenting with complaints of worsening shortness of breath. The physician notes bilateral basilar rales on
auscultation of the lungs. The physician also notes peripheral edema in both of the lower extremities. After pressing
firmly over the patient’s right anterior medial tibia for 5 seconds, the physician notes to the scribe that there is a 6mm
indentation that disappears after just over a minute. What grade should this scribe document for this patient’s
peripheral edema?

A. 0
B. +1
C. +2
D. +3
E. +4
A physician is working with a scribe while they are performing a cardiac exam on a 55 year old male. The patient is
presenting with complaints of worsening shortness of breath. The physician notes bilateral basilar rales on
auscultation of the lungs. The physician also notes peripheral edema in both of the lower extremities. After pressing
firmly over the patient’s right anterior medial tibia for 5 seconds, the physician notes to the scribe that there is a 6mm
indentation that disappears after just over a minute. What grade should this scribe document for this patient’s
peripheral edema?

A. 0 An easy way to approximate


B. +1 the appropriate grading is to
C. +2 divide the amount of mm
D. +3 indentation by two
E. +4
Ex: 6 mm indentation/2 = +3
grading
A medical student is taking the HPI of a 35 year old female complaining of chest pain. The patient describes the pain
as sharp, and notes that it is made worse when they cough, swallow, or take a deep breath. It improves when the
patient sits or leans forward. When auscultating the chest, the student hears a high-pitched scratching type noise
over the left sternal border while the patient was sitting up and leaning forward. What anterior Chapman’s point
could this student check for that would best correlate with this patient’s history and physical exam findings?

A. 2nd Intercostal space, proximal to


the sternum
B. Superior to the left clavicle
C. 6th intercostal space on the left
D. Tip of the 12th rib
A medical student is taking the HPI of a 35 year old female complaining of chest pain. The patient describes the pain
as sharp, and notes that it is made worse when they cough, swallow, or take a deep breath. It improves when the
patient sits or leans forward. When auscultating the chest, the student hears a high-pitched scratching type noise
over the left sternal border while the patient was sitting up and leaning forward. What anterior Chapman’s point
could this student check for that would best correlate with this patient’s history and physical exam findings?

A. 2nd Intercostal space, proximal to


the sternum
B. Superior to the left clavicle
C. 6th intercostal space on the left
D. Tip of the 12th rib

The patient’s history and physical exam


findings are consistent with myocarditis.
That is why this specific anterior Chapman
point may be found, as this condition
involves the myocardium.
A 27 year old male presents to a clinic with complaints of a cough that has lasted for 3 weeks. When
performing a Osteopathic examination of the patients ribs, the physician notes that ribs 4-8 on the right
have an exhalation dysfunction. What rib is considered the key rib to treat if the physician were to
perform rib MET on this patient?

A. The 8th rib


B. The 6th rib
C. The 4th rib
D. Both the 4th and 8th rib are key ribs
A 27 year old male presents to a clinic with complaints of a cough that has lasted for 3 weeks. When
performing a Osteopathic examination of the patients ribs, the physician notes that ribs 4-8 on the right
have an exhalation dysfunction. What rib is considered the key rib to treat if the physician were to
perform rib MET on this patient?

A. The 8th rib


B. The 6th rib
C. The 4th rib
D. Both the 4th and 8th rib are key ribs
A 60 year old female presents with complaints shortness of breath. As a part of the evaluation of this
patient, the physician notes that ribs 2-6 on the left don’t seem to move as well as well as those on the
right while the patient inhales. If the physician were to perform rib MET on this patient, which of the of
the following would be included as a key part of this treatment?

A. The physician would exaggerate the patient’s


exhalation by pressing inferiorly on the patients
2nd rib, and then resist motion while the patient
inhales
B. The patient inhales deeply, holds their breath, and
then would contract their posterior scalene while
the physician resists this motion
C. The patient inhales deeply, holds their breath, and
then would contract their serratus anterior while
the physician resists this motion
D. The physician would exaggerate the patient’s
exhalation by pressing inferiorly on the patient’s
6th rib, and then resist motion while the patient
inhales
A 60 year old female presents with complaints shortness of breath. As a part of the evaluation of this
patient, the physician notes that ribs 2-6 on the left don’t seem to move as well as well as those on the
right while the patient inhales. If the physician were to perform rib MET on this patient, which of the of
the following would be included as a key part of this treatment?

A. The physician would exaggerate the patient’s


exhalation by pressing inferiorly on the patients
2nd rib, and then resist motion while the patient
inhales
B. The patient inhales deeply, holds their breath, and See the next slide for an explanation
then would contract their posterior scalene while
the physician resists this motion
C. The patient inhales deeply, holds their breath, and
then would contract their serratus anterior while
the physician resists this motion
D. The physician would exaggerate the patient’s
exhalation by pressing inferiorly on the patient’s
6th rib, and then resist motion while the patient
inhales
To answer this question, you need to first recognize that the patient has an
exhalation dysfunction. Then remember that you must treat the top rib of an
exhaled group first. Then you must remember that the treatment for rib two
(the top rib in this case) is to contract the posterior scalene against resistance.

For ribs 1 and 2, remember:

“It’s annoying to get up at 1 A.M 2 Pee” →

Rib 1 = anterior and medial scalene

Rib 2 = posterior scalene


Best of luck! Please let me know if you have
any questions.

Parker Howe - parker.howe@kansascity.edu


Question 1

78 y/o male comes into the office for his annual check up. Upon palpation, you find that the PMI is smaller
than it should be. During auscultation you hear a murmur at the mitral valve. Pitting edema is at +3. If you
were to consider a cardiac pathology, which of the following findings would be appropriate?

A. Hypertonicity of paraspinal muscles of T1-4


B. Hypertonicity of paraspinal muscles T1-6
C. Hypertonicity of paraspinal muscles at T5-10
D. Hypertonicity of paraspinal muscles at T10 - L2
Question 1 LO3: Palpation

78 y/o male comes into the office for his annual check up. Upon palpation, you find that the PMI is smaller
than it should be. During auscultation you hear a murmur at the mitral valve. Pitting edema is at +3. If you
were to consider a cardiac pathology, which of the following findings would be appropriate?

A. Hypertonicity of paraspinal muscles of T1-4


B. Hypertonicity of paraspinal muscles T1-6
a. KNOW KNOW KNOW KNOW KNOW THE VISCEROSOMATICS! THEY WILL COME UP TIME AND TIME
AGAIN!
C. Hypertonicity of paraspinal muscles at T5-10
D. Hypertonicity of paraspinal muscles at T10 - L2
Question 2 matching

Mitral Valve Right 2nd ICS at sternal border

Aortic Valve Left 4th ICS at sternal border

Pulmonic Valve Left 2nd ICS at sternal border

Tricuspid Valve Left 5th ICS at the midclavicular line

Point of Maximal Impulse Left 5th ICS at the midclavicular line


Question 2 matching:
LO3 Auscultation and Palpation

Mitral Valve Right 2nd ICS at sternal border

Aortic Valve Left 4th ICS at sternal border

Pulmonic Valve Left 2nd ICS at sternal border

Tricuspid Valve Left 5th ICS at the midclavicular line

Point of Maximal Impulse Left 5th ICS at the midclavicular line


Question 3

During a cardiac exam, you suspect that there might be some right sided heart failure due to the left sided
heart failure. You found that the distance above the sternal angle to be 4 cm H20. What is the total JVP?
And is this abnormal?

A. 4cm H20
B. 6cm H20
C. 9cm H20
D. 12cm H20
1. Yes
2. No
Question 3: LO3 and 7

During a cardiac exam, you suspect that there might be some right sided heart failure due to the left sided heart
failure. You found that the distance above the sternal angle to be 4 cm H20. What is the total JVP? And is this
abnormal?

A. 4cm H20
B. 6cm H20
C. 9cm H20
a. You add 5 cm to what you found
D. 12cm H20
1. Yes
2. No
a. Normal is 6-8
b. Anything 9 and above is considered abnormal and should indicate further testing
Question 4

Patient presents with pain in the chest wall with breathing. You’ve rule out all other life threatening factors and
have determined it to be an MSK issue. Upon palpation you’ve realized that ribs 2-6 are having issues moving into
exhalation. Which of the following describes the appropriate technique?

A. The patient’s upper body is gently flexed and sidebent towards the somatic dysfunction and the rib is
depressed from above
B. The patient is sidebent to the level of the somatic dysfunction and the rib is depressed from above
C. The patient lies prone and the rib is pushed upward
D. Use the Post Scalene muscle
E. Use the Serratus Anterior Muscle
Question 4: LO5

Patient presents with pain in the chest wall with breathing. You’ve rule out all other life threatening factors and have
determined it to be an MSK issue. Upon palpation you’ve realized that ribs 2-6 are having issues moving into exhalation.
Which of the following describes the appropriate technique?

A. The patient’s upper body is gently flexed and sidebent towards the somatic dysfunction and the rib is depressed from
above
B. The patient is sidebent to the level of the somatic dysfunction and the rib is depressed from above
a. The somatic dysfunction with Rib 2-6 Inhaled SD
b. BITE: Treat the key rib → Rib 6
c. You do not use the muscles in Inhalation somatic dysfunctions
d. KNOW your set ups and how it changes from SD to SD
C. The patient lies prone and the rib is pushed upward
D. Use the Post Scalene muscle
E. Use the Serratus Anterior Muscle
Question 5

A 25 y/o patient comes into the office with chest pain that localizes to the right side. He is having troubles
breathing in. After excluding life threatening causes, you have found it to be a MSK issue. Ribs 4-12 are
having restriction in motion with exhalation. Which of the following describes the proper setup?

A. The patient's head is rotated approx 30 degrees away from the affected rib and the dorsal surface
of the wrist of the SD side is placed on the forehead
B. The patient raises the arm of the dysfunctional side and places the hand over the head
C. The patient’s shoulder is flexed 90 degrees and reaches for the other shoulder
D. The patient’s shoulder is abducted to 90 degrees
E. The patient is prone
Question 5: LO5

A 25 y/o patient comes into the office with chest pain that localizes to the right side. He is having troubles breathing in. After
excluding life threatening causes, you have found it to be a MSK issue. Ribs 4-12 are having restriction in motion with
exhalation. Which of the following describes the proper setup?

A. The patient's head is rotated approx 30 degrees away from the affected rib and the dorsal surface of the wrist of the
SD side is placed on the forehead : Rib 1-2
B. The patient raises the arm of the dysfunctional side and places the hand over the head : Rib 3-5
C. The patient’s shoulder is flexed 90 degrees and reaches for the other shoulder : Rib 6-8
D. The patient’s shoulder is abducted to 90 degrees : Rib 9-10
E. The patient is prone
a. Key rib is rib 12 (BITE)
b. Rib 4-12 inhalation SD
c. Make sure you know the set up for each one. It will be useful in the practical and on the exam
d. Know the muscles that are going to be involved
Please reach out if you have any questions
Victor.Phan@kansascity.edu
Approach to Lower Respiratory
Complaint and Rib HVLA &
Counterstrain Lab
By Spencer Phillips & Lauren Chi
Approach to the Lower Respiratory & Rib HVLA/CS

1. Where is the Chapman Point located that denotes the location of the Lower Lung?

A. 1st Intercostal Space


B. 2nd Intercostal Space
C. 3rd Intercostal Space
D. 4th Intercostal Space
E. 5th Intercostal Space
Approach to the Lower Respiratory & Rib HVLA/CS

1. Where is the Chapman Point located that denotes the location of the Lower Lung?

A. 1st Intercostal Space


B. 2nd Intercostal Space
C. 3rd Intercostal Space
D. 4th Intercostal Space
E. 5th Intercostal Space
Approach to the Lower Respiratory & Rib HVLA/CS

2. You have just started your first day of clerkship and third year rotations. Congratulations, you have
completed your didactic learning! On your first day you are rounding with a nurse doing physical exams
when he notes that there is stridor in the lungs of a 5 year old child. Which of the following is your best
next course of action?

A. Proceed with an ENT exam, this child likely has a sinus infection
B. I don’t know, this sounds like something I would learn in second year
C. Stridor is normal in a 5 year old
D. Stridor is a medical emergency and needs to be addressed STAT!
E. Stridor is a fancy medical word for clear sounding lungs, right?
Approach to the Lower Respiratory & Rib HVLA/CS

2. You have just started your first day of clerkship and third year rotations. Congratulations, you have
completed your didactic learning! On your first day you are rounding with a nurse doing physical exams
when he notes that there is stridor in the lungs of a 5 year old child. Which of the following is your best
next course of action?

A. Proceed with an ENT exam, this child likely has a sinus infection
B. I don’t know, this sounds like something I would learn in second year
C. Stridor is normal in a 5 year old
D. Stridor is a medical emergency and needs to be addressed STAT!
E. Stridor is a fancy medical word for clear sounding lungs, right?
Approach to the Lower Respiratory & Rib HVLA/CS

3. Your attending asks you to come over and listen to the patient as they perform the Egophony specialty
test. As they perform they test you hear a distinct “A” sound. Which of the following best explains this?

A. This is a normal sounding test result


B. This is a clinical finding exclusive to tuberculosis infection
C. This is a clinical finding indicating abnormal consolidations
D. This is a clinical finding seen in Upper Respiratory Pathologies
Approach to the Lower Respiratory & Rib HVLA/CS

3. Your attending asks you to come over and listen to the patient as they perform the Egophony specialty
test. As they perform they test you hear a distinct “A” sound. Which of the following best explains this?

A. This is a normal sounding test result


B. This is a clinical finding exclusive to tuberculosis infection
C. This is a clinical finding indicating abnormal consolidations
D. This is a clinical finding seen in Upper Respiratory Pathologies
Approach to the Lower Respiratory & Rib HVLA/CS

4. After completing a physical exam of the ribs you identify that Ribs 3-6 have an inhalational SD. Which
of the following is true?

A. HVLA is effective here by driving the rib superior


B. HVLA is effective here by driving the rib inferior
C. There is no treatment with HVLA of ribs 3-10
D. I would focus my treatment on rib 3.
Approach to the Lower Respiratory & Rib HVLA/CS

4. After completing a physical exam of the ribs you identify that Ribs 3-6 have an inhalational SD. Which
of the following is true?

A. HVLA is effective here by driving the rib superior


B. HVLA is effective here by driving the rib inferior
C. There is no treatment with HVLA of ribs 3-10
D. I would focus my treatment on rib 3.
Approach to the Lower Respiratory & Rib HVLA/CS

5. With the help of your attending, you discover a tenderpoint on a patient located at PR1. What would be
the approximate treatment position of this tenderpoint with counterstrain?

A. F-StRT
B. F-StRA
C. f-F ST RT
D. E SA Rt
E. F SA RA
Approach to the Lower Respiratory & Rib HVLA/CS

5. With the help of your attending, you discover a tenderpoint on a patient located at PR1. What would be
the approximate treatment position of this tenderpoint with counterstrain?

A. F-StRT
B. F-StRA
C. f-F ST RT
D. E SA Rt
E. F SA RA
6. Match the Percussion Sound Description with the Pathologic Condition

1. Flat
A. COPD, pneumothorax
2. Dull
B. Large pleural effusion
3. Resonant
C. Large pneumothorax
4. Hyperresonant
D. Simple chronic bronchitis
5. Tympanic E. Lobar pneumonia
6. Match the Percussion Sound Description with the Pathologic Condition

1. Flat A. COPD, pneumothorax


2. Dull B. Large pleural effusion
3. Resonant C. Large pneumothorax
4. Hyperresonant D. Simple chronic bronchitis
5. Tympanic E. Lobar pneumonia

1. B, 2. E, 3. D, 4. A, 5. C
Percussion Review
7. A group of kids (Eren, Armin, and Mikasa) come to your OMM clinic looking absolutely distressed. They
have been through a lot, but now Eren has a fever, cough, and rapid, shallow breathing. You decide to use the
specialty test Tactile Fremitus.

What are abnormal findings would you suspect for a Eren?

A. “ee” sounds like “A” and has a nasal bleating quality


B. Sounds are louder
C. Increased: Clear, strong vibration
D. Whispered sounds are louder, clearer
7. A group of kids (Eren, Armin, and Mikasa) come to your OMM clinic looking absolutely distressed.
They have been through a lot, but now Eren has a fever, cough, and rapid, shallow breathing. You
decide to use the specialty test Tactile Fremitus.

A. “ee” sounds like “A” and has a nasal bleating quality - Egophony
B. Sounds are louder - bronchophony
C. Increased: Clear, strong vibration - Signs of Pneumonia
a. Note: Decreased: little or no vibration → Increases suspicion of COPD
D. Whispered sounds are louder, clearer - Whispered Pectoriloquy
8. A 26 year old male (he/him) comes to your clinic complaining of chest pain. He says he was sparring with his students (Naruto,
Sasuke, and Sakura) all of last week. This is his week off and would like to enjoy his time reading his very interesting novels. You
assess and don’t find any signs of fractured ribs. However, you do find ribs 3-10 prefer exhalation.

Which of the following is correct for a Rib 3-10 Exhalation SD, Supine HVLA?

A. Doctor places thenar eminence of the caudad hand posterior and inferior to angle of dysfunctional rib.
At the end of the exhalation, doctor applies a posterior thrust directed slightly caudal to physician's thenar eminence
B. Doctor places thenar eminence of the caudad hand posterior and inferior to the angle of the dysfunctional rib.
At the end of exhalation, doctor applies a posterior thrust directed slightly caudal to the physician's thenar eminence
C. Sidebending the head toward the dysfunctional rib and rotating away until motion is felt at T1 or T2.
At the end of exhalation, on inhaled rib, thrust inferiorly and medially towards mid inferior pectoralis. ~45º inferior and
medial, slightly anterior.
8. A 26 year old male (he/him) comes to your clinic complaining of chest pain. He says he was sparring with his students (Naruto,
Sasuke, and Sakura) all of last week. This is his week off and would like to enjoy his time reading his very interesting novels. You
assess and don’t find any signs of fractured ribs. However, you do find ribs 3-10 prefer exhalation.

Which of the following is correct for a Rib 3-10 Exhalation SD, Supine HVLA?

A. Doctor places thenar eminence of the caudad hand posterior and inferior to angle of dysfunctional rib.
At the end of the exhalation, doctor applies a posterior thrust directed slightly caudal to physician's thenar eminence
B. Doctor places the thenar eminence of the caudad hand posterior and superior to the angle of the dysfunctional rib.
At the end of exhalation, doctor applies a posterior thrust directed superior to the thenar eminence.
a. Rib 3-10 Inhalation SD, Supine HVLA
C. Sidebending the head toward the dysfunctional rib and rotating away until motion is felt at T1 or T2.
At the end of exhalation, on inhaled rib, thrust inferiorly and medially towards mid inferior pectoralis. ~45º inferior and
medial, slightly anterior.
a. Seated Rib 1-2, Inhalation SD, HVLA
9. Next on your schedule, you have a male high school volleyball team here at you OMM clinic ready to receive OMM
treatment before their next big tournament. You are assessing Hinata when you suddenly find Ribs 11 and 12
Inhalation SD.

For a Rib 11,12 inhalation Prone HVLA, what is the direction of the activating force?

a. At the end of exhalation, deliver an anterior HVLA thrust to the rib


b. At the end of exhalation, deliver an medial and caudal thrust to the rib
c. At the end of exhalation, deliver an medial and cephalad thrust to the rib
d. At the end of exhalation, deliver a lateral and caudal HVLA thrust to the rib
e. At the end of exhalation, deliver a lateral and cephalad HVLA thrust to the rib
9. Next on your schedule, you have a male high school volleyball team here at you OMM
clinic ready to receive OMM treatment before their next big tournament. You are
assessing Hinata when you suddenly find Ribs 11 and 12 Inhalation SD.

For a Rib 11,12 inhalation Prone HVLA, what is the direction of the activating force?

a. At the end of exhalation, deliver an anterior HVLA thrust to the rib


b. At the end of exhalation, deliver an medial and caudal thrust to the rib
c. At the end of exhalation, deliver an medial and cephalad thrust to the rib
d. At the end of exhalation, deliver a lateral and caudal HVLA thrust to the rib
e. At the end of exhalation, deliver a lateral and cephalad HVLA thrust to the rib
10. Name the correct treatment position for each tender point:

1. Anterior Ribs 1 & 2


2. Anterior Ribs 3-10
3. Posterior Rib 1
4. Posterior Rib 2
5. Posterior Ribs 3-10
10. Name the correct treatment position for each tender point:

1. Anterior Ribs 1 & 2: FSRT


2. Anterior Ribs 3-10: f-F RST
3. Posterior Rib 1 - eSaRt-T (Maverick)!
4. Posterior Rib 2 - FSARA
5. Posterior Ribs 3-10 - FSARA
Good Luck! You can do it!

Lauren Chi (lauren.chi@kansascity.edu)

You might also like